Главная Юзердоски Каталог Трекер NSFW Настройки

Космос и астрономия

Ответить в тред Ответить в тред
Check this out!
<<
Назад | Вниз | Каталог | Обновить | Автообновление | 552 78 170
Тред тупых вопросов №181 Аноним 06/08/23 Вск 08:25:34 760386 1
PlutoinTrueColo[...].jpg 215Кб, 1024x1024
1024x1024
94ac4ee0b636e3e[...].jpg 90Кб, 1000x532
1000x532
image5645afde8e[...].jpg 126Кб, 1200x831
1200x831
Novyie-Gorizont[...].jpg 682Кб, 1980x815
1980x815
Тред вопросов о жизни, Вселенной и всем таком.
Спрашиваем то, за что в других местах выдают путевку в биореактор. Здесь анонимные учёные мирового уровня критически рассмотрят любые гениальные идеи и нарисованные в Paint схемы.

Прошлый тред >>757854 (OP)
Аноним 06/08/23 Вск 12:05:59 760397 2
На что опирается гироскоп? На материальную среду?
Аноним 06/08/23 Вск 12:15:06 760400 3
>>760397
На инерцию, довен
Аноним 06/08/23 Вск 12:48:16 760401 4
>>760400
Откуда берется инерция?
Аноним 06/08/23 Вск 13:39:06 760402 5
Может ли быть снизмассивная чёрная дыра, которая при размере песчинки весила бы как два мешка цемента? Ну какая-нибудь карманная чёрная дыра
Аноним 06/08/23 Вск 14:24:38 760406 6
>>760402
Гипотетически - да. Механизмов естественного образования не придумали, кроме первичных при большом взрыве, но таких не замечали пока.
Аноним 06/08/23 Вск 14:38:36 760407 7
>>760402
Возможно даже ещё снизмассивней были, которые как один мешок цемента весили. Но они в 2 раза менее вероятней, чем те, что 2 мешка цемента весили.
Аноним 06/08/23 Вск 19:46:19 760418 8
>>760402
Она испарится за миллионы лет
Аноним 06/08/23 Вск 23:45:26 760424 9
Астрономы ещё до нашей эры могли наблюдать попятное движение Марса. Это подразумевает, что уже тогда могли вычислять местоположение (координаты) небесного тела с точностью, недоступной человеческому глазу. С помощью каких приборов производились эти наблюдения - астролябии, секстанты и т.д.? Посоветуйте пожалуйста книги, описывающие эволюцию подобных инструментов и их принципы работы.

Какие инструменты для определения координат небесных тел сегодня есть в относительной доступности (примерно также, как телескопы для любителей)?
Аноним 06/08/23 Вск 23:59:12 760426 10
>>760401
От начального момента крутящего.
Аноним 07/08/23 Пнд 00:32:45 760427 11
>>760397
на корпус смартфона
Аноним 07/08/23 Пнд 03:51:51 760438 12
земля протыкает космос своей ракетой словно жопу пассивного педераста хуй, или же наоборот, космос жадно всасывает в себя ракету словно хуй дрочёный в жопу пидора?

помогите разобраться. с уважением.
Аноним 07/08/23 Пнд 05:45:42 760440 13
image.png 99Кб, 236x281
236x281
>>760438
А ты знаешь что такое нюанс?
Аноним 07/08/23 Пнд 15:53:40 760448 14
>>760426
А у чего может быть крутящий момент?
Аноним 07/08/23 Пнд 15:58:47 760449 15
Аноним 07/08/23 Пнд 16:05:57 760450 16
>>760449
Мо-ло-де-ц! А из чего сделан механизм?
Аноним 07/08/23 Пнд 17:24:28 760453 17
>>760450
Из материи.
Материя опирается на материю
Аноним 07/08/23 Пнд 18:06:20 760458 18
>>760453
А если гироскоп в вакууме?
Аноним 07/08/23 Пнд 20:42:24 760461 19
>>760458
На вакуум опирается, что тут непонятного?
Вакуум это же пространство, которое искривляется от материальных тел. То есть пространство физически взаимодействует с материальными телами и даже оказывает на них усилие в виде гравитации.
То есть вакуум может порождать усилие, воздействующее на материальные тела.
Вот когда гироскоп вращается - он закручивает вакуум и опирается на вакуум - что тут непонятного?
Аноним 07/08/23 Пнд 21:25:41 760467 20
>>760461
Как можно вакуум закрутить?
Аноним 07/08/23 Пнд 22:37:20 760471 21
>>760467
>Как можно вакуум закрутить?
Абсолютно также - как тела искривляют вакуум - вызывая взаимную гравитацию.
Вакуум он же жидкий!
Аноним 07/08/23 Пнд 22:51:31 760472 22
Что такое "поляризация вакуума"?
Аноним 07/08/23 Пнд 23:02:04 760473 23
>>760472
> Что такое "поляризация вакуума"?
ты когда нибудь радугу видел?
Аноним 07/08/23 Пнд 23:20:51 760474 24
>>760471
Подожди, вот у тебя вращается гироскоп в вакууме. Вакум вокруг закрутился. И теперь мы в этот закрученый вакум что-то пихаем и оно должно крутиться вместе с вакуумом, но оно не крутится. Значит что?
Аноним 07/08/23 Пнд 23:25:36 760476 25
>>760474
>вакум
Вакуум
фикс
Аноним 07/08/23 Пнд 23:57:50 760477 26
>>760450
Из пластиков и металлов, т.е. из хим соединений.
Аноним 08/08/23 Втр 00:04:41 760478 27
image.png 64Кб, 320x180
320x180
>>760477
А из чего сотворены ёбаные пластики, металлы,(а ещё неметаллы и органические поеботы, типа "ацетон") тоесь химичнические соединения? А , блят?
Аноним 08/08/23 Втр 01:37:26 760481 28
>>760478
Из молекул, атомов, субатомных частиц, всяких струн бран или полей.
Аноним 08/08/23 Втр 05:35:03 760484 29
>>760481
Блят, какие нахуй браны-хуяны? Ты говна поел? Струны-хуюны. Тебя Юрик лоза в жопу штоли выебал?
Аноним 08/08/23 Втр 11:45:12 760490 30
>>760472
Квантовые флуктуации полей в вакууме, которые проявляются в постоянном спонтанном рождении и уничтожении пар частиц и античастиц
Аноним 08/08/23 Втр 12:08:29 760491 31
Аноним 08/08/23 Втр 13:27:52 760495 32
>>760472
Когда у полей становится настолько высокая напряженность, что выплывают их квантовые или порционные свойства, что формируют целый ряд феноменов.
Строго говоря многие поля и так поляризованы и формируют вещество, а те что нет формируют излучение, пример тут электромагнитное поле.
Обычно под поляризацией вакуума понимается качественный переход электромагнитного поля и его смешение с "электронным" полем.
Аноним 08/08/23 Втр 20:02:03 760503 33
Аноним 08/08/23 Втр 20:52:35 760505 34
5f760c20c868360[...].jpg 112Кб, 736x736
736x736
Если мы возмём чёрную дыру с радиусом 1000м и покроем её решёткой на высоте 1005м, то есть на 5м выше горизонта событий, то значит ли это, что мы тем самым блокируем гравитацию и падающее в ЧД тело?
Подобно тому, как мы можем скинуть куриное яйцо с высоты вытянутой руки человека на балконе на 9 этаже и дать ему упасть на землю, чтобы кинетическая энергия яйца при падении его и уничтожила, или подставить другу рукую на высоте вытянутой прямо руки, чтобы яйцо упало в руку, не ускорилось и не самоуничтожилось об асфальт, так и внешняя защита на ЧД спасёт от смерти под горизонтом событий
Аноним 08/08/23 Втр 21:20:15 760506 35
>>760505
Этот аналог неправилен в контексте чёрных дыр, так как не учитывает сложную и необычную структуру пространства-времени около горизонта событий чёрной дыры.

Гравитация: Гравитационное притяжение чёрной дыры так велико, что на расстоянии всего в нескольких метрах от горизонта событий оно существенно искривляет пространство-время. Такое искривление делает невозможным просто "заблокировать" гравитационное притяжение решёткой или другим материальным объектом. Сама решётка будет сильно притягиваться к чёрной дыре.

Прочность материалов: Для создания какой-либо структуры, способной удерживать объекты от падения в чёрную дыру, потребуется материал с абсолютно невероятными механическими свойствами. В условиях такого сильного гравитационного поля любой известный материал не сможет удержать форму и будет немедленно разрушен.

Динамика падения: Падающее тело будет нарастать в скорости, приближаясь к горизонту событий, и никакая решётка не сможет его остановить, если она не имеет невероятной прочности и устойчивости. Также, если мы рассматриваем действие общей теории относительности, само пространство-время около горизонта событий будет "течь" в сторону чёрной дыры, что сделает остановку падения ещё более непосильной задачей.

В итоге, попытка "заблокировать" гравитационное притяжение чёрной дыры или предотвратить падение объекта в неё с помощью решётки или любой другой структуры недействительна. Это не соответствует нашему пониманию физики около чёрных дыр и принципов общей теории относительности.
Аноним 08/08/23 Втр 21:32:23 760507 36
Почему в плотноприжатых кубиках невозможно движение?
Аноним 08/08/23 Втр 21:32:43 760508 37
Парни, как вы относитесь к супердетерминизму (Bell) ?

Почему считается в эксперименте по неравенствам Белла, что "план частиц может иметь только 9 вариантов и всё строится на логике теория вероятностей макромира, что почему то "подтверждает", что частица там уже должна была лежать с определенным спином по всем осям, а не быть рэндомной?
А что если есть скрытые параметры, которые программируют именно "квантовое поведение" (через этаж Реальности), чтобы выдавать квантовый ответ, чтобы получалось 50 на 50? Почему план должен быть таким элементарным, что если я вверх, то ты вниз, ты вниз, то я вверх, и вот у тебя 9 вариантов... и получается вероятности и в общем Белл прав, распишитесь. расходимся.

Что за навязывание шаблонов теории вероятнстей макромира на квантовый мир?
Своим макро-шампуром мне к глюон тыкаешь.
Аноним 08/08/23 Втр 21:34:58 760509 38
>>760508
Те, кто отрицают детерминизм - чмошники. Ещё вопросы?
Аноним 08/08/23 Втр 21:38:06 760511 39
>>760490
в каком пту учился?
Аноним 08/08/23 Втр 21:38:39 760512 40
image.png 1066Кб, 1000x1000
1000x1000
>>760506
>Гравитация: Гравитационное притяжение чёрной дыры так велико, что на расстоянии всего в нескольких метрах от горизонта событий оно существенно искривляет пространство-время. Такое искривление делает невозможным просто "заблокировать" гравитационное притяжение решёткой или другим материальным объектом. Сама решётка будет сильно притягиваться к чёрной дыре.
> Также, если мы рассматриваем действие общей теории относительности, само пространство-время около горизонта событий будет "течь" в сторону чёрной дыры, что сделает остановку падения ещё более непосильной задачей.
Значит, не на высоте 5 метров, а 50, чтобы быть выше планки существенности этих проблемТакже, если мы рассматриваем действие общей теории относительности, само пространство-время около горизонта событий будет "течь" в сторону чёрной дыры, что сделает остановку падения ещё более непосильной задачей.
> Прочность материалов: Для создания какой-либо структуры, способной удерживать объекты от падения в чёрную дыру, потребуется материал с абсолютно невероятными механическими свойствами. В условиях такого сильного гравитационного поля любой известный материал не сможет удержать форму и будет немедленно разрушен.
Тогда не решётку, а монолитный - и самособой полый - металлошар (или кварцешар, или зубошар, или алмазошар), укреплённый углеродными трубками, баллестическим жиром и бронёй "чешуя дракона"
Аноним 08/08/23 Втр 21:43:37 760514 41
>>760512
Даже с увеличенным расстоянием и более продвинутой конструкцией, задача по-прежнему остается чрезвычайно сложной и, скорее всего, нереализуемой с нашим текущим уровнем технологий и пониманием физики.

У ультрамассивных чёрных дыр градиент гравитации на горизонте событий намного слабее, чем у меньших чёрных дыр. Это означает, что разница в силе тяжести между двумя точками, разделёнными небольшим расстоянием (например, двумя концами вашей конструкции), будет меньше. В теории это делает идею чуть более осуществимой.

Тем не менее, остаются серьезные проблемы:

Стабильность: Удержание конструкции на стабильной орбите вблизи горизонта событий всё равно потребует огромных усилий и энергии. Возможно, нужно будет использовать активные системы управления, чтобы удерживать её в правильном положении.

Прочность материалов: Несмотря на меньший градиент гравитации, по-прежнему потребуются материалы и конструкции, выходящие за рамки текущих технологий.

Действие гравитации: Чёрная дыра всё равно будет оказывать огромное гравитационное влияние на всё, что находится вблизи, и это должно быть учтено при планировании любых манипуляций в этой области.

Динамика вблизи горизонта событий: Вблизи горизонта событий действуют сложные и неинтуитивные законы физики, которые могут привести к непредсказуемым и трудным для управления эффектам.

Построение и обслуживание: Даже если такую конструкцию можно было бы построить, её постройка и обслуживание были бы исключительно сложными и дорогостоящими задачами, возможно, выходящими за рамки текущих человеческих возможностей.
Аноним 08/08/23 Втр 22:03:38 760515 42
Что значит "гравитация не квантуется"?
Аноним 08/08/23 Втр 22:09:28 760517 43
>>760515
Квантуется. Не сама гравитация, а гравитационное поле. Но не точно.
Аноним 08/08/23 Втр 23:36:14 760520 44
>>760484
Адекват! Бранодебилы полные дауны, у них там реально секта струнных теологов который уже нихуя не соображт в своих выдуманных хуевертях
Аноним 09/08/23 Срд 07:55:44 760524 45
>>760505
С любой реальной "решеткой" это не сработает по очевидным, не связанным с относительностью причинам. Масса такой дыры порядка 0,3 солнечной и ускорение свободного падения возле ее горизонта событий исчисляется триллионами g. Тут и любая решетка моментально схлопнеться и уж любой падающий на нее предмет ее снесет и не заметит. То есть нужны всякие материалы с "бесконечной прочностью" которые отвязывают любой мысленный эксперимент от реальности.
Аноним 09/08/23 Срд 10:16:35 760526 46
>>760515
Под квантованием понимается как некоторая непрерывная система меняется дискретным образом. Для частиц вроде электронов просто. Они проявляют себя только скачками - пресловутое измерение, в остальное время они существуют в своей особенной реальности. Правила перехода из их собственной реальности в нашу и есть квантование.
С полями такая же история, но сложнее, ибо поле это по сути бесконечномерная частица, другими словами поле можно интерпретировать бесконечными способами. Короче в этом произволе надо как-то ориентировать, а иначе не описать все это и построить модель.
Вот тут приходят различные математические трюки, которые убирают этот произвол, что позволяет простоит стройную модель
Одно из этих преставление поля как совокупность рождающихся-уничтожающийся частиц. Этот метод оказался очень продуктивным, что стал стандартным. И когда говоря квантование, подразумевают квантование именно по этому методу. Но у этого способа есть недостатки, он не работает не на всех масштабах и не на всех полях.
Гравитация как поле по этому методу не квантуется.
Есть и другие методы квантования полей, но у них свои проблемы.

Это все хуйня, ОТОшная гравитация все таки не поле, а геометрия по сути. И описание дискретных скачков, при сохранение непрерывных геометрических структур до сих не разрешенная задача.
Аноним 09/08/23 Срд 10:36:42 760527 47
>>760511
>пук
>>760508
>как вы относитесь к супердетерминизму (Bell) ?
Антинаучная хуйня и говно без задач
>>760515
Это значит, что не существует теории квантовой гравитации и гравитацию невозможно описывать в терминах частиц
Аноним 09/08/23 Срд 14:53:45 760531 48
>>760386 (OP)
> всерьёз обсуждают чд
> когда недавнее исследование показало, что то, что в центре ведёт себя не как чд и ничего в себя не затягивает

космач 20 23, итоги
Аноним 09/08/23 Срд 15:26:36 760533 49
Аноним 09/08/23 Срд 15:28:07 760534 50
>>760533
> А что в центре?
Кремль, Красная площадь, ГУМ.
Аноним 09/08/23 Срд 15:58:05 760535 51
сап двач, план построить дом из вулкана в солнце, плюс паруса в новое солнце и отправить колонию на марс, помогите, хелп.
Аноним 09/08/23 Срд 18:54:09 760536 52
Аноним 09/08/23 Срд 21:54:23 760540 53
>>760531
> ничего в себя не затягивает
Прям как любая другая ЧД, нихуя ж себе. Иди учи азы астрономии чтоб не обсираться на такой дилетантской хуете.
Аноним 09/08/23 Срд 21:56:43 760541 54
На планетах с плотной, как на Земле атмосферой, но с существенно меньшей, чем у Земли, массы с гравитацией (скажем, в пределах до марсовой) могут ли существовать смерчи, торнадо и аналогичные погодные явления?
>>760531
>> когда недавнее исследование показало, что то, что в центре ведёт себя не как чд и ничего в себя не затягивает
А ну-ка, пруфы в студию!
Аноним 09/08/23 Срд 22:23:00 760544 55
>>760541
Да. На Марсе же есть.
Аноним 10/08/23 Чтв 00:00:17 760552 56
А какая минимальная плотность среды для распространения звука? А то слышал несколько шизов говорили что якобы в космосе может быть звук так как он не совсем пустой(пара атомов на кубический метр ага) просто его НЕ СЛЫШНО.
Аноним 10/08/23 Чтв 00:02:52 760553 57
>>760461
Ой мля это же плотноприжатый хуесос не кормите...
Аноним 10/08/23 Чтв 00:03:10 760554 58
Пространство состоит из кубиков?
Аноним 10/08/23 Чтв 00:06:51 760555 59
>>760553
Из чего состоит пространство?
Аноним 10/08/23 Чтв 00:07:43 760556 60
>>760505
Хароший вопрос, но с черной дырой слишком сложно вот моя вэрсия поясните аноны! Если вдоль экватора построить мост вокруг всей планеты, а потом одновременно убрать все опоры то мост будет парить? Он ведь со всех сторон одинаково притягивается, значит притяжение с одной стороны будет отталкивать с другой и все такое. Допустим материал суперпрочный и он не сломается.
Аноним 10/08/23 Чтв 00:12:33 760557 61
>>760555
Пространство это то, что заполняет мой у тебя во рту.
Аноним 10/08/23 Чтв 00:13:12 760558 62
>>760556
Быстро равновесие потеряет и пизданется.
Аноним 10/08/23 Чтв 00:13:24 760559 63
Аноним 10/08/23 Чтв 02:14:37 760560 64
Поясните за выражение
Современный ПК в миллион раз мощнее ПК на котором летали на луну
Аноним 10/08/23 Чтв 07:23:09 760564 65
>>760560
Чего тут непонятного то? Мощность измеряется в ваттах. Если современная "игровая" пекарня жрет допустим до 500Вт, то в миллион раз меньше будет 0.5мВт. А это примерно как микроконтроллер какой-нибудь потребляет, если тактовую частоту ему пониже сделать.
Аноним 10/08/23 Чтв 07:53:39 760566 66
>>760560
12 тысяч операций с плавающей точкой в секунду у аполло
против
12 миллиардов операций у Core Quad 9400 (который вышел почти 15 лет назад)
Современный пека более чем в миллион раз мощнее.
Аноним 10/08/23 Чтв 08:44:37 760569 67
>>760566
Мощность измеряется в ваттах, а не в количестве операций в секунду, поэтому ты вообще не в тему ответил.

А даже если сравнивать количество операций в секунду, то стоит учитывать что для интела это получено вместе с SSE и всеми ядрами, а далеко не все задачи параллелятся. Иначе можно было сравнивать это с производительностью GPU, что явно глупо.
Аноним 10/08/23 Чтв 10:18:04 760570 68
Аноним 10/08/23 Чтв 10:49:13 760571 69
>>760552
>что якобы в космосе может быть звук так как он не совсем пустой(пара атомов на кубический метр ага) просто его НЕ СЛЫШНО
Они правы
Аноним 10/08/23 Чтв 11:14:02 760575 70
>>760569
Вычислительная мощность. Вычисляется в флопсах (операциях с плавающей точкой в секунду).

Мимо
Аноним 10/08/23 Чтв 11:42:21 760577 71
>>760552
Под звуком понимается две разные вещи, одно это собственно физическое явление, другое это наше восприятие посредством органов слуха.
В первом случае главный критерий это длина свободного пробега частиц среды. И тут играет роль не плотность среды как таковая, а концентрация частиц и их взаимодействие с друг другом. Частицами могут быть легкие электроны и протоны с крайне низкой концентрацией(чуть выше чем в радиолампах), но при этом хорошо переносить звук почти как обычный воздух. А могут быть тяжелые молекулы и пылинки, которые почти не передают звук. Пример реголит.
Аноним 10/08/23 Чтв 12:00:50 760579 72
>>760575
>операциях с плавающей точкой в секунду
А если у нас нет аппаратной плавающей запятой, например не нужно оказалось и потому не сделали, то тогда все, хуевый компьютер?
Ну и опять же, сравни количество этих операций для GPU и CPU.
Аноним 10/08/23 Чтв 14:24:36 760582 73
>>760566
> 12 тысяч операций с плавающей точкой в секунду у Аполло
+ 2 килобайта оперативы на ферритовых кольцах.
Но что-то пошло не так и при самом первом прилунении комп зафакапился, садиться пришлось вручную джойстиком, по телевизору с торчавшим вниз видиконом с объективом.
Оперативка на ферритовых кольцах юзалась аж до середины 70х, где-нибудь и позже.
Аноним 10/08/23 Чтв 14:26:33 760583 74
>>760579
Зависит от задач, которые выполняет данных компухтер. Но сейчас на компухтеры накладываются задачи, где без плавающей точкой не обойтись. особенно важно для всякой мобильной хуиты, где важна обработка сигналов риалтайм. Так что да, хуевый компьютер.
Некорректно сравнивать вычислительные мощности ГПУ и ЦПУ. ГПУ заточены на сугубо узкие задачи(в общем сводится к переумножению матриц) и заветные флопсы условны. В других задачах ГПУ сильно слабее ЦПУ.
Аноним 10/08/23 Чтв 14:55:40 760585 75
>>760579
> например не нужно оказалось и потому не сделали, то тогда все, хуевый компьютер?
"Не нужно оказалось" - такого не было, а было "не влезает в заданую площадь чипа чтобы получалось требуемое число чипов с пластины, поэтому будет отдельный чип FPU-сопроцессора для тех кому нужно быстрее пересчитывать таблицы".
10/08/23 Чтв 16:33:26 760589 76
001.jpg 174Кб, 1200x658
1200x658
002.jpg 391Кб, 2047x806
2047x806
>>760575
>Вычислительная мощность. Вычисляется в флопсах (операциях с плавающей точкой в секунду).
Флопсы это косвенный критерий. Он не учитывает аппаратную специализацию. При одних и тех же флопсах CPU и GPU будут решать одну задачу за сильно разное время. Но не все задачи, впихуемые в CPU, можно впихнуть в GPU.

Также следует отметить, что ядро процессора не "знает", что такое float. Оно умеет только складывать и вычитать 32/64 битные числа. В сущности это такой счётчик, функционально подобный ещё механической "паскалине" 1642 года. А вот всякие ветвления, обработка плавающей точки и прочие высокоуровневые абстракции возможны благодаря инструкциям процессора. И вот они уже у каждого процессора могут быть разные. И в зависимости от бохатства функционала этих инструкций операцию с float можно выполнить за разное количество тактов (причём для сложения, умножения, деления и корня они тоже будут разные).

Срач на тему инструкций процессора будет идти вечно. Одни говорят, что слишком много инструкций плохо, потому что ебанёшься под это всё компиляторы писать и выводить эти сложные часы с кукушкой на рынок. Другие говорят, что слишком много инструкций хорошо, потому что можно делать ёба-хаки на ассемблере и корень вычислять с первого такта.

Поэтому вообще непонятно, как вычислительные мощности разных архитектур сравнивать. Можно считать такты, транзисторы, флопсы в секунду, инструкции, параллельные потоки, кэш, и результат в каждом случае будет разным. А в будущем ещё и нейроморфные процессоры завезут, и там вообще заебёшься сравнивать это всё.
Аноним 10/08/23 Чтв 16:41:16 760590 77
>>760552
>А какая минимальная плотность среды для распространения звука? А то слышал несколько шизов говорили что якобы в космосе может быть звук так как он не совсем пустой(пара атомов на кубический метр ага) просто его НЕ СЛЫШНО.
Слышно, просто тебе медведь в детстве на ухо наступил, и ты не слышишь звуки ниже 16Гц. А при длине свободного пробега частиц межзвёздной (и тем более межгалактической) среды уместнее говорить о микрогерцах или даже наногерцах. И какой-нибудь ёбаный хтонический мегамонстр звёздных размеров мог бы их даже услышать.
Аноним 10/08/23 Чтв 17:41:37 760596 78
>>760474
>Подожди, вот у тебя вращается гироскоп в вакууме.
Да

> Вакум вокруг закрутился.
Ага - он же жидкий и вакуум искривляется от тел - вызывая гравитацию

> И теперь мы в этот закрученый вакум что-то пихаем и оно должно крутиться вместе с вакуумом, но оно не крутится.
Хуйню несешь - у тебя же гироскоп вращается в закрученном вакууме и опирается на закрученный вакуум
Ты хочешь что-то впихнуть в гироскоп? - Ну наверное это что-то тоже закрутится вместе с гироскопом - почему нет?
Аноним 10/08/23 Чтв 18:20:19 760599 79
>>760589
Тащемта сам формат float (как и производные вроде double) появился из существующей аппаратной реализации тогдашних суперкомпьютеров.
Уже позже, когда появилась идея идеология RISC и ее экстремальные проявления, выкинули плавающую точку в отдельный блок.
Впрочем в конце 0-ых Intel умудрились сделать мутанта суперскалярную архитектуру для арифметики с плавающей точкой, да еще в векторным представлением. И ядро современных процессоров вполне очень даже знает, что такое float.
Аноним 10/08/23 Чтв 18:23:21 760600 80
Кстати, на поверхности нейтронных звезд может собраться больцмановский мозг? Как он будет выглядеть?
Аноним 10/08/23 Чтв 18:33:21 760601 81
Бамп
Аноним 10/08/23 Чтв 18:40:43 760604 82
>>760596
>Ты хочешь что-то впихнуть в гироскоп?
В вакуум рядом с гироскопом.
Аноним 10/08/23 Чтв 19:15:06 760606 83
>>760604
Ну так впихни сперва что-то рядом с искривленным вакуумом вокруг тел и посмотри - искривится ли предмет или нет.
Аноним 10/08/23 Чтв 20:16:39 760610 84
image.png 1711Кб, 1285x793
1285x793
>>760600
>Кстати, на поверхности нейтронных звезд может собраться больцмановский мозг? Как он будет выглядеть?
ПЛОТНОПРИЖАТЫМ
Аноним 10/08/23 Чтв 20:35:24 760611 85
>>760560
На луну летали на пк? Запустили елите на спектуруме и полетели на луну? лолбля
Аноним 10/08/23 Чтв 20:37:19 760612 86
image.png 246Кб, 600x450
600x450
Аноним 10/08/23 Чтв 21:38:02 760614 87
Аноним 10/08/23 Чтв 21:56:36 760615 88
Чисто технически, может ли человек сейчас безвозвратно запулить 1-10 людей на орбиту Юпитера и дальше (живых).
Аноним 10/08/23 Чтв 22:10:44 760618 89
>>760556
>суперпрочный
Вот в этом и всё дело, что такого суперпрочного материала не бывает. Как и вечного мотора.
Аноним 10/08/23 Чтв 22:16:24 760620 90
>>760579
>А если у нас нет аппаратной плавающей запятой
Мумулируется как нехуй, определяешь мантиссу и операции, сколько там, десяток на сложение будет вместо одной у тех кто может в флоп нативно?
И закономерный слив по перфомансу.
Аноним 10/08/23 Чтв 22:32:07 760622 91
>>760620
>Мумулируется
Реализуется программно, а не это. А во вторых плавающая запятая это для неосиляторов фиксированной запятой.
Например в цифровой обработке сигналов запросто можно обойтись без плавающей запятой, а цифровая обработка сигналов может быть весьма затратной по ресурсам. Это я к тому что глупо мерить в плавающей запятой, оно не везде нужно.
Аноним 10/08/23 Чтв 22:40:53 760623 92
>>760622
Аналоговые компы когда? Сколько можно усираться этими жалкими аппроксимациями, давно бы уже
Аноним 10/08/23 Чтв 22:47:58 760624 93
>>760615
Чисто технически чтобы долететь до орбиты Марса и вернуться - нужно поднять и разогнать 800 тонн, но это по оценкам самих космонавтов, ирл - накидывай ещё 1000 тонн на более жырную защиту от солнечных вспышек чтобы не сидеть сутки напролёт в тесной защитной капсуле.
До Юпитера наверно можно удваивать вторую массу.
Но двихателя нет, ни на первое, ни на второе.
Аноним 11/08/23 Птн 00:07:50 760629 94
>>760624
Ну раз такой умный, то отчего не летишь?
Аноним 11/08/23 Птн 00:19:02 760630 95
Стикер 191Кб, 510x510
510x510
>>760629
Я не могу поднять 800 тонн.
Аноним 11/08/23 Птн 00:46:02 760631 96
Почему не развивают сборку всякого оборудования и целых кораблей на орбите? Допустим запустили большую йобу частями маленькими ракетами и собрали её на орбите. И не нужно Сатурн 5 заново изобретать.
Аноним 11/08/23 Птн 02:56:49 760659 97
>>760631
Потому же, почему самодельные машины и самолёты запрещены. Рептилоиды не велят, мало ли чего ты там насобираешь.
Аноним 11/08/23 Птн 04:14:55 760664 98
Сори за тупой вопрос, но не знаю где еще спросить
Короче, к сути, можете насоветовать каналов в телеге на всякую космическую/научную тематику, типа с запусками ракет всякими, фотками с хабблов всяких и в целом новостей по поводу обследования/освоения космоса и в этом ключе. Можно не научпоперские а узкоспециализированные, не обязательно на русском, можно и на инглише. Есть ли такие каналы в целом? Заранее спасибо.
Аноним 11/08/23 Птн 06:29:43 760668 99
>>760664
>в телеге
Ну здравствуйте, Дмитрий Олегович
Аноним 11/08/23 Птн 06:33:50 760669 100
>>760659
>Потому же, почему самодельные машины и самолёты запрещены. Рептилоиды не велят, мало ли чего ты там насобираешь.
Так нейтральные воды есть - организуешь там всемирный двачерский космодром и запускаешь ракеты на Марс - в чём проблема? А по поводу, что рептилии уничтожат - так ты защищайся.
Кишка тонка? Мозгов мало? Лентяй залупский? Руки из жопы? - ну пшел нахер тогда, ничтожество.
Аноним 11/08/23 Птн 08:37:41 760680 101
>>760541
Не будет пруфов, исследование есть как и ссылка, но пусть оно всплывёт в самый неприятный для местных кривопросьранственных шизиков момент

Я всё это написал чтобы подготовить их, они как видишь всё цепляются за эту херню

Но что ещё смешнее, что раз тут никто из вас про это исследование не в курсе, то это значит, что тут настоящих учёных нет, одни клоуны, так что добавьте в тред пометку, что тут только шизики верцны, застрявшие в середине века
Аноним 11/08/23 Птн 09:01:41 760682 102
>>760680
>что тут настоящих учёных нет
А ты настоящий в говне моченый?
Аноним 11/08/23 Птн 10:25:19 760691 103
Можно ли считать черную дыру принципиально неуязвимым и всеразрушающим объектом? Имаджин: небольшая, массой в несколько солнц, ЧД, которая разумна каким то образом управляется разумной сверхцивилизацией может менять направление своего движения. Что ей смогут противопоставить хоть человечество, хоть раса анунаков со сферой Дайсона, кроме как непрерывно оформлять съебатора, надеясь, что не догонит, и ожидая миллионы лет, пока испарится?
Аноним 11/08/23 Птн 10:31:33 760694 104
>>760691
>Можно ли считать черную дыру принципиально неуязвимым
Нет. Все черные дыры рано или поздно испарятся
Аноним 11/08/23 Птн 11:08:51 760703 105
>>760694
>Все черные дыры рано или поздно испарятся
Скорее поздно чем рано, а до тех пор делает омномномном. И кстати, что при таких вводных мешает дыре набирать массу быстрее, чем испаряться?
Аноним 11/08/23 Птн 11:52:56 760715 106
>>760682
> в говне моченый

Как же ты себя не уважаешь, позорник

Лучше за новыми публикациями следил, тогда б не заосрял тред фантазиями навроде этих

> Можно ли считать черную дыру принципиально неуязвимым и всеразрушающим объектом
> Все черные дыры рано или поздно испарятся

Это уже не наука, а уровень плоской Земли
Аноним 11/08/23 Птн 12:41:03 760722 107
>>760703
Много факторов, но многие из которых сводиться к моменту вращению, который надо как-то сбрасывать для падения вещество под горизонт. На вращающийся ЧД даже свету тяжело упасть. Наблюдаемая аккреция вещества только сильнее раскручивает ЧД, что сильно ограничивает темп набора массы. Собственно проблема откуда в галактиках сверхмассивные ЧД, они даже сейчас не могут отъесться на миллиард солнечных масс.
Аноним 11/08/23 Птн 12:53:57 760726 108
>>760722
Хм, симс легит.
А при испарении ведь момент уменьшается? Тогда все равно получается, что ЧД не сдохнет до тех пор, пока ей будет что жрать вокруг.
Аноним 11/08/23 Птн 13:45:22 760739 109
>>760726
Нет, не уменьшает. Вообще большой вопрос, что происходит с ЧД малой массы на последних стадиях испарениях. Чем меньше ЧД тех хуже она способна сбросить свой момент вращения и рано или поздно вращение станет критическим и горизонт перестанет скрывать сингулярность.
Если ЧД не является часть гравитационно связанной системы, то как максимум чем она может питаться это релятивистскими фотонами и то очень не эффективно, ибо поглощение и испарение придут быстро в термодинамическое равновесие.
Аноним 11/08/23 Птн 15:05:17 760742 110
Аноним 11/08/23 Птн 20:34:35 760778 111
>>760582
> + 2 килобайта оперативы на ферритовых кольцах.
Вообще-то 4
Аноним 11/08/23 Птн 21:44:33 760786 112
>>760778
Вообще-то 2 килослова, где слово это 15 бит + 1 паритет.
Аноним 11/08/23 Птн 21:56:23 760790 113
>>760630
Ничего, анончик, это нормально, я тута попрбувал полтонны поднять - спинку сорвал, до сих пор лечусь((((
Аноним 11/08/23 Птн 22:00:22 760791 114
>>760631
Для такого дела надо лет 10 стандарты и госты всякие изобретать, а если эта жопа ещё и международная (это лол бля было бы) - то вавилонская башня - хоть и ложь да в ней намёк, бля) Вроде скайлэб с союзом не стыковалась же? Разные же "половые органы"
Аноним 12/08/23 Суб 07:18:19 760808 115
Парни. Масса гнет пространство время и тем самым вызывает гравитацию. Так?
Масса я это по большей части энергия глюонов и кварков.
Можно ли сказать что сильное взаимодействие и есть граитация?
Тогда в мире лишь 3 взаимодействия, а не 4. Так?
Аноним 12/08/23 Суб 08:07:21 760811 116
>>760808
>Масса гнет пространство время и тем самым вызывает гравитацию. Так?
Нет, не та так. Тебе нужно знать аж 16 параметров и их нельзя вывести только из массы, чтоб знать как материя гнет пространство-время.
>Можно ли сказать что сильное взаимодействие и есть гравитация?
Нет, нельзя. Они совершенно разные.
Аноним 12/08/23 Суб 08:26:50 760816 117
>>760808
Нет, гравитация на безмассовые частицы переносчики взаимодействия тоже действует.
Проблема в определении взаимодействий, гравитация сильно не похожа на остальные два с половиной.
Аноним 12/08/23 Суб 08:30:48 760818 118
Маск недавно случайно стартовый стол, а вслед за ним - и саму ракету.

В связи с этим: насколько реализуемо и экономически оправдано выглядит воздушный старт? Нет, не запуск с самолетов - подозреваю, что все известные не смогут поднять в воздух йобу даже для лунной миссии, не говоря уже о марсианской. А как насчет одноразовых аэростатов? Самых примитивных, лишь бы не лопнули от порывов ветра, на водороде. Снизу на тросах подвешена ракета. Ей позволяется мотыляться как угодно, да и сам шар тоже может сносить куда угодно. В нужный момент и на нужной высоте включается зажигание, при наборе достаточной тяги все тросы сбрасываются, ракета висит на тяге и медленно начинает разгон, шар как более парусный объект улетает нахуй вверх и в сторону, можно его еще взорвать по команде, чтобы сразу сгорел и не налепился на ракету.
Преимущества: полное устранение рикошета от выхлопа, экологичность по выхлопу, вообще безопасный для Земли запуск, если что-то пошло не так, небольшой бонус к потенциальной энергии ракеты, отсутствие затрат на стартовый стол.

Недостатки: затраты на аэростат, понадобится пересчет начального участка траектории в реальном времени, понадобится разработать ракету и ее подвес так, чтобы она не разорвалась под собственной тяжестью, ведь до сих пор прочность считали больше на сжатие, чем на разрыв.

Какие еще подкаменные воды?
Аноним 12/08/23 Суб 08:44:17 760819 119
>>760818
Добавлю, что если ракету запускать с небольшой высоты, всего несколько км - то один еростат можно заменить слаженной работой нескольких десятков грузовых вертолетов, тут уже полная многоразовость и отказоустойчивость, которые в тренде.
Аноним 12/08/23 Суб 09:04:02 760820 120
>>760818
Платина. Аэростаты пиздос какие массивные, ракеты не поднимут.
Посчитай сам, чтоб на слово не верить, возьми подъемную силу гелия, можешь водорода, все равно охереешь, посчитай нужный объем, посчитай сколько надо материала для сферического коня в вакууме, ну ты понял. Полезно и познавательно.
Аноним 12/08/23 Суб 09:44:03 760821 121
>>760808
>Можно ли сказать что сильное взаимодействие и есть граитация?
Гравитация - это геометрический эффект искривления пространства-времени, которое создается любой массой/энергией. Пространство-время искривляют и фотоны - переносчики электромагнитного взаимодействия.
Вся суть в том, что гравитация - это не отдельная сила, а чисто геометрические причуды пространства-времени, реагирующего таким образом на воздействие со стороны различных масс/энергий
Аноним 12/08/23 Суб 09:58:56 760822 122
>>760820
Считать лень, поэтому возьму оче грубую прикидку.
Гинденбург поднимал 100 тонн груза. 200к кубометров водорода.
Союз-2.1б весит 300 тонн.

При этом Гинденбург - это полноценное судно с жестким каркасом из стали, оболочкой из не самой легкой прорезиненной ткани, с движками, топливом и прочей байдой, не относящейся к полезной нагрузке.
А у нас единственное требование - не порваться от порывов ветра и внутреннего давления.
Арамидная ткань как основной и практически единственный конструкционный материал стоит не слишком дорого, весит гораздо меньше, выдерживает огромные нагрузки на разрыв.

Итого, 400к кубов водорода в мягкой оболочке диаметром 90 м - достаточно с большим запасом.

Чо, по прежнему технологии сверхцивилизаций прошлого, или делать йоба пуски с бетоном, бункерами, газоотводами и фермами гораздо выгоднее?

В таком случае давай разберем по частям идею с вертолетами.
Аноним 12/08/23 Суб 10:23:53 760824 123
>>760822
Напомню очевидное, что с увеличением линейных размеров подъемная сила растет в кубе, а парусность, необходимая прочность, и следовательно, масса оболочки - в квадрате, так что соотношение массы конструкции к массе полезного груза стремительно падает и будет заведомо меньше единицы при таких размерах.
Аноним 12/08/23 Суб 10:44:49 760825 124
>>760822
Давай помогу.
Плотность водорода 0.08375 кг на кубометр.
Плотность воздуха 1.225 кг на кубометр на уровне моря.
На десяти километрах 0.4135 килограмма на кубометр.
Кубометр водорода может поднять примерно 300 грамм на высоту около 10 километров.
Для 300 тонн надо миллион кубометров, или шар более 60 метров радиусом. И пару миллионов на заправку этого шара водородом.
Ну вроде можно, казалось бы?
А теперь всплывают вопросы сопромата, который я считать не буду, но суть про то что оболочка которая потащит 300 тонн будет сама нифига не слабой, утечки водорода, флуктуации атмосферы, сильная зависимость от погоды в целом, горючесть водорода, и абсолютно хуевейший выхлоп от воздушного старта в целом, тебе примерно такой же мощности стартовую массу РКН надо иметь что и при старте с земли, главное - скорость набирать, жалкие 10 километров высоты погоды не играют почти никак! Подумой почему аэростаты и дирижопли в целом вышли из оборота, почему самолеты так их вытеснили, и станет понятнее.
А, ну и в огурцов погоняй, но с РСС чтобы было заметно какая разница будет от старта с высоты.
Аноним 12/08/23 Суб 12:12:47 760839 125
image.png 1151Кб, 800x800
800x800
Посоветуйте если уж не хорошую, годную, то хотя бы просто неплохую, приемлемую космофантастику, причём именно СОВЕТСКУЮ космофантастику.
Читнул "Аэлиту" Толстого и - внезапное открытие этого года для меня лично - "На оранжевой планете" Леонида Оношко, посмотрел "Планету Бурь" 1961 года... и что-то как-то проникся этой наивной т доброй советской фантастикой, где вместо ксеноморфов и злых корпораций, новый мир и приключенческие исследования.
Вы знаете ещё что-нибудь подобного? Очень прошу, поделитесь названиями
Аноним 12/08/23 Суб 12:42:11 760845 126
В обсерваториях можно ночевать?
Вон вроде обещали с 11 по 13 августа дождь Персемиды. В окно посмотрел: тучи мешают. Это так машину надо иметь... палатку и все такое, чем в жизни не занимался.
31 августа суперлуну голубую обещают еще...
Аноним 12/08/23 Суб 12:49:52 760846 127
>>760610
Адекват! Вопрос на засыпку, а что соберётся на обратной стороне нейтронной звезды?
Аноним 12/08/23 Суб 13:25:35 760849 128
Почему в космосе такие большие расстояния? Какой размер у вселенной был спустя 10 секунд, год, 10 лет после бэнга?
Аноним 12/08/23 Суб 14:53:27 760853 129
Письменность появилась примерно 5800 лет до нашей эры. Почему людишки так долго развивались? Что мешало?
Аноним 12/08/23 Суб 14:59:40 760856 130
>>760853
>долго
В масштабах космоса тысячелетия это наносекунды. С точки зрения возраста Вселенной мы только что возникли, а уже бороздим космос.
Аноним 12/08/23 Суб 15:18:09 760858 131
>>760853
А могла и вовсе не возникнуть еще миллион лет. А может еще миллион лет назад возникла и канула в небытие когда макаки не выкупили профит. Получилось то что получилось, каких-то особых причин не вижу.
Аноним 12/08/23 Суб 15:41:44 760860 132
>>760849
>Почему в космосе такие большие расстояния?
Потому что Вселенная постоянно расширяется
Аноним 12/08/23 Суб 15:44:11 760861 133
Новая модель предсказывает, что Вселенная в 2 раза старше, чем сейчас общепринято считать

Новое исследование канадских учёных предполагает, что возраст Вселенной составляет 26,7 миллиарда лет, а это ставит под сомнение общепринятую оценку в 13,7 миллиарда лет, основанную на модели Lambda-CDM.

Модель Lambda-CDM (Lambda-Cold Dark Matter), более известная как «стандартная космологическая модель» — широко принятая модель, описывающая эволюцию Вселенной с самых ранних этапов до наших дней. В течение многих лет астрономы и физики рассчитывали возраст нашей Вселенной на основе данных о красном смещении света, исходящего от далёких галактик. В 2021 году на основе этой модели возраст Вселенной был оценён в 13,797 миллиарда лет.

Новое исследование бросает вызов доминирующей космологической модели и проливает новый свет на так называемую «проблему ранних галактик».


Мнение?
Аноним 12/08/23 Суб 15:47:32 760862 134
>>760856
>В масштабах космоса тысячелетия это наносекунды
Не нано, а микро. Разница огромная.
Аноним 12/08/23 Суб 16:04:51 760865 135
Аноним 12/08/23 Суб 21:13:29 760881 136
>>760825
Спасибо за расчет.

Теперь главное.
Основная идея была не в этом жалком десятке километров. А в том, что воздушный старт не требует специально подготовленного стартового стола и потенциально гораздо более безопасен для ракеты, если вместо стола - масковская бетонка или вообще пересеченная местность.
Другой вариант - подводный старт, но у меня есть подозрение, что струи кипящей морской воды и пара опасны для массивной ракеты даже больше, чем грунт.

>почему аэростаты и дирижопли в целом вышли из оборота, почему самолеты так их вытеснили
Хуевая управляемость, низкая скорость, проблемы с хранением, при небольших размерах - недостаточная полезная нагрузка, при больших - экономия на набор и удержание высоты съедается тратами на преодоление сопротивления воздуха/ветра.
Все это к конкретному рассматриваемому вопросу почти не относится.

Но ок, допустим, что это слишком жирно и опасно - делать одноразовый водородный шарик для каждого запуска.

Что насчет 20 вертолетов Ми-26 с тросами, которые поднимут ракету на 2 км перед запуском?
Затраты исключительно на ТО и топливо на десятки минут, в течении которых будет произведен запуск.
Потом машины можно использовать для других нужд.
Система отказоустойчива - даже если одна-две-три машины наебнутся с аварийным отцеплением троса, остальные удержат груз.
Если наебнется сама ракета - у пилотов есть время съебаться в стороны от огненного шара.
Аноним 12/08/23 Суб 21:23:25 760882 137
>>760881
>Что насчет 20 вертолетов Ми-26 с тросами, которые поднимут ракету на 2 км перед запуском?
Хрень.
Езжай значит в Антарктиду и прихвати с собой ядерный реактор.
Доезжаешь до самого южного полюса, начинаешь в сторонке топить лед, потом водичку подаешь на систему распыления, которая должна образовать горку высотой 100 км. Потом система тросов - и подъемник до Космоса готов!
Аноним 12/08/23 Суб 21:26:11 760883 138
Ну как вариант - можно построить горку из ледяных кубиков лего.
Аноним 12/08/23 Суб 21:41:50 760885 139
>>760818
>Маск недавно случайно стартовый стол, а вслед за ним - и саму ракету.
>В связи с этим: насколько реализуемо и экономически оправдано выглядит воздушный старт? Нет, не запуск с самолетов - подозреваю, что все известные не смогут поднять в воздух йобу даже для лунной миссии, не говоря уже о марсианской. А как насчет одноразовых аэростатов?
И я, и мемный дед Бояршинов эту тему неоднократно разбирали.
Тезисно: чем меньше ракета, тем больше её ебут аэродинамические потери. Для ракеты весом порядка тонны эти потери соизмеримы с дельтой, требующейся для выхода на орбиту. Поэтому сверхмалые ракеты запускать с самолётов и стратостатов можно и нужно.

Для тяжёлых ракет это неактуально. Лобовое сопротивление по кубу-квадрату растёт медленнее массы, и для ракеты размером с Протон аэродинамические потери составляют единичные проценты.

Что касается Маска и его огромной ебалды, то такую ебалду поднимать в воздух нечем. Тут аэростат нужен размером с 50 гинденбургов. Проще его гендерно-небинарным инженерам пизды дать и сказать, чтобы скопировали стартовую площадку №110 для Н-1/Энергии с нормальным бетониумом и газоотводами.
Аноним 12/08/23 Суб 21:55:58 760887 140
Чтоб посрать на луне, нужно изобрести скафандр для говна?
Аноним 12/08/23 Суб 22:28:49 760892 141
image.png 269Кб, 704x293
704x293
>>760881
>воздушный старт не требует специально подготовленного стартового стола
Ага, он требует чуть ли не большего по размерам и сложности комплекса стартовых сооружений И специально подготовленной ракеты.
А еще знаешь какая веселость есть? Если вдруг надо скрабнуть, то ты просто не включаешь движки и ракета остается на столе и ее увозят. С аэростатом уже не скрабнешь, запустил в полет - уже запускать движки надо, иначе никак.
>Другой вариант - подводный старт, но у меня есть подозрение, что струи кипящей морской воды и пара опасны для массивной ракеты даже больше, чем грунт.
Нет, они лучше чем куски грунта, но ракету тоже надо специальную. Хорошая затея была большую тупую ракету, стали похую на часок-другой в водичке, если ее потом в ней же и утопят после разделения.
>Но ок, допустим, что это слишком жирно и опасно - делать одноразовый водородный шарик для каждого запуска.
Да как, тут жир и опас примерно ракетного уровня, это ерунда. Это просто усложнение с кучей новых параметров котоыре могут пойти не так и не дают никакого ощутимого профита.
>Что насчет 20 вертолетов Ми-26 с тросами, которые поднимут ракету на 2 км перед запуском?
А ничего. Это возможно, но какой профит в подобной ебатне?
Аноним 12/08/23 Суб 23:59:35 760898 142
Без названия (3[...].jpg 119Кб, 1559x864
1559x864
>>760892
>какой профит в подобной ебатне
Да все тот же, епта.
Экономия на стартовой площадке. Запуск практически из любого места. Да хоть с поезда, который привозит ракету. Хоть с баржи. Хоть с неподготовленной поверхности Марса. Хоть прямо там, где ракету собрали. Нужны только цистерны с компонентами и заправщики.

Если вдруг надо скрабнуть - внезапно, на еростатах существуют клапана, а как эту сложнейшую операцию провести с хеликоптерами - предлагаю придумать самостоятельно.
Аноним 13/08/23 Вск 00:02:15 760899 143
malyiy-sputnik-[...].jpg 63Кб, 1440x870
1440x870
Сап, космач! Вопрос немного не в тему, но всё-таки космический. А дело вот в чём:

Я поступил в универ на специальность "ракетно космические комплексы и космонавтика" и теперь в течении недели мне нужно определится с направлением подготовки. На выбор даётся два профиля:

1) Ракетные транспортные системы
(учат проектировать и собирать ракеты, ракетные двигатели, рассчитывать баллистические траектории запуска и тому подобное) учится тут трудно, зато работать интереснее и можно смотреть на запуски своих ракет и гордиться проделанным трудом.

2) Малогабаритные космические аппараты
(тут учат конструировать космические аппараты, преимущественно наноспутники, собирать и программировать их электронику) тут учится гораздо проще, но, как по мне, готовые спутники не приносят столько же радости как пуск ракеты. Поэтому я боюсь, что в будущем данная работа превратится в РАБоту.

Проблема в том, что оба направления мне одинаково интересны и имеют одинаковое количество плюсов и минусов, поэтому я не могу выбрать одно из них. Так что хочу узнать, что бы выбрали аноны на моём месте, может это поможет мне определиться.
Аноним 13/08/23 Вск 00:40:01 760901 144
>>760821
Именно, гравитация и искривления пространства времени по причине присутствия энергии - это уже следствие, а не фундаментальный закон.
Аноним 13/08/23 Вск 06:58:53 760904 145
>>760899
>Ракетные транспортные системы
>работать интереснее и можно смотреть на запуски своих ракет и гордиться проделанным трудом.
Ну давай, присаживайся и загибая пальцы вспоминай новые ракеты, которые спроектировали и научили летать в РФ:
1)
2)
3)
4)
5)
Аноним 13/08/23 Вск 07:16:30 760905 146
>>760898
>Экономия на стартовой площадке.
Стоимость лётного часа 20 Ми-26 имаджинируешь?
Аноним 13/08/23 Вск 08:06:04 760906 147
>>760898
>Экономия на стартовой площадке.
Илон, ты не на том экономишь. Стартовый стол - это лишь небольшая часть обширной стартовой инфраструктуры, и если ты вместо стартового стола будешь ебашить вертодром или инфраструктуру для шаров с поддержкой ракеты под это дело, выйдет наверняка даже дороже, заправлять и поддерживать ракету все равно надо.
>Запуск практически из любого места.
Это не такая проблема как кажется. Над населенными местами все равно трасса не может проходить, потому стартовать можно из ограниченного количества мест на суше или около. Так что может йоба-баржу со стартовым столом глянешь вместо?
>Да хоть с поезда, который привозит ракету.
А это уже существует, если что. Не РКН, правда, но конверсии никто не запрещал.
>Хоть с баржи.
А это вполне может быть перспективной затеей. Для крупных ракету сильно надежнее и проще воздушного старта.
>Хоть с неподготовленной поверхности Марса.
Ну тут вертолеты и шары определенно не помогут, так что или учиться копать газоотводы роботами, или бетоний удобный разрабатывать.
>Нужны только цистерны с компонентами и заправщики.
Как и для стола? А про инфраструктуру для вертолетов ты забыл, или они необслуживаемые?
>Если вдруг надо скрабнуть - внезапно, на еростатах существуют клапана
Стравливаешь, ракета бахается о землю и взрывается к хуям. Или стравливаешь и ракету тоже, и поливаешь керосином и/или криогеникой окрестности и ракета все равно ломается оземь, потому что хуй тебе а не на неконтроллируемом аэростате мягкая контроллируемая посадка.
> а как эту сложнейшую операцию провести с хеликоптерами - предлагаю придумать самостоятельно.
У ркеты (первой ступени) есть посадочные ноги? Нахуя лишний весь везешь?
Нету? Тогда как ты на землю не сломав движки или корпус вертолетами уложишь?
Ракеты хрупкие дохуя.
Аноним 13/08/23 Вск 08:36:59 760908 148
PGM-11RedstoneC[...].jpg 36Кб, 544x704
544x704
>>760898
>>760898
>Экономия на стартовой площадке. Запуск практически из любого места.
Главная проблема сторонников воздушного старта - он базируется на сравнении "картинок в голове". С одной стороны у нас в голове картинка ебического стартового комплекса с огромной башней обслуживания, тысячами тонн бетона под ним и грамадный краулер который с трудом везет туда ракету. С другой стороны у нас в голове картинка ракеты закрепленной под крылом самолета или подцепленой на тросе под вертолетом или аэростатом которая легко так отцепляется, двигатели делают пыщь и она летит на орбиту. Вот только те ракеты которые легко закрепить на самолет или под аэростат и потом легко отцепить и запустить, с земли можно пускать даже с табуреточки и обслуживать небольшим краном. Пикрелейтед устанока ракеты стартовой массой в 27 тонн, пускай и боевой. Ракету стартовой массой в 60 тонн можно запускать даже со специально подготовленного грузовика. Это если что намек на конверсионный Тополь.
Собственно тут в треде даже немного проглядывается - начали с "возможного решения проблем Старшипа" но почему то считаем аэростат для ракеты не на 5000 тонн а всего на 300.
Аноним 13/08/23 Вск 09:27:56 760909 149
image (3).jpg 17Кб, 1200x676
1200x676
755508548101228.jpeg 588Кб, 1180x730
1180x730
1603695337bajko[...].jpg 233Кб, 640x453
640x453
70251452365e0b5[...].jpg 78Кб, 710x381
710x381
>>760905
>Стоимость лётного часа 20 Ми-26 имаджинируешь?
Все равно это суммы разных порядков. И кстати в идеале это будет именно час, если не меньше - когда ракета готова к старту и должна быть поднята.

>>760906
>Илон, ты не на том экономишь
Забавно будет, если его референты почитывают двач, и вскоре он начнет работу над одной из этих охуительных идей. Тут надо быть осторожным, смотря что для нас важнее: эпичный попил бабла и обсер конкурента Роскосмоса, или космический прогресс всего человечества.

>Не РКН, правда, но конверсии никто не запрещал.
Я в курсе про БЖРК. Разница в выхлопе, как основном негативном факторе - порядок, если не два. Тут не конверсия, тут полный дестрой платформы и жд путей. Рассматривал этот вариант только в совокупности с вертолетами.

>А это вполне может быть перспективной затеей.
Опять же, или 1) с вертолетами, или 2) подводный старт, где специально обученная ракета делает бултых, разворачивается жопой вниз благодаря остойчивости, а стартует только когда баржа уплывет на безопасное расстояние. Если тупо с самой баржи - помашем ей ручкой. Если в барже сделать технологическую газоотводную дырень - ебейшие потоки пара и газа меняют плотность воды - и снова косплей Титаника.

>или учиться копать газоотводы роботами, или бетоний удобный разрабатывать
Считаем затраты времени и средств, что по итогу выгоднее. Идите нахуй, мыши, я стратег

>инфраструктуру для вертолетов
Считаем, что выгоднее - держать вертодром или перебросить вертолеты из не столь отдаленных мест постоянного базирования.

поливаешь керосином и/или криогеникой окрестности
Керосин гидразин можно и откачать. Криогенику стравить экологически безопасно, если не ебанет не должно

>посадочные ноги
Нинужны, можно и на бок класть, главное осторожненько и в какую нибудь старто-посадочную колыбельку из натяжных тросов, потому что >хрупкие дохуя

Короче дискасс вплотную приблизился к необходимости сопромат-экономических расчетов, с чем и поздравляю.


Пикрил - последствия хлопка с задымлением в воздухе, и на земле.
Аноним 13/08/23 Вск 09:43:56 760910 150
image.png 425Кб, 730x473
730x473
image.png 483Кб, 1280x720
1280x720
image.png 205Кб, 640x480
640x480
>>760909
>Забавно будет, если его референты почитывают двач, и вскоре он начнет работу над одной из этих охуительных идей. Тут надо быть осторожным, смотря что для нас важнее: эпичный попил бабла и обсер конкурента Роскосмоса, или космический прогресс всего человечества.
Отож, еще как читает, а ты думаешь почему до сих пор огурцов на марс не заслал как обещал? Аноны напомнили что он про радиацию забыл!
>Я в курсе про БЖРК. Разница в выхлопе, как основном негативном факторе - порядок, если не два. Тут не конверсия, тут полный дестрой платформы и жд путей. Рассматривал этот вариант только в совокупности с вертолетами.
Это как вообще? Прилетают вертолеты, цепляют кучу тросов, стараясь не задеть друг дружку и потом вытаскивают ракету из вагона?
>Опять же, или 1) с вертолетами, или 2) подводный старт, где специально обученная ракета делает бултых, разворачивается жопой вниз благодаря остойчивости, а стартует только когда баржа уплывет на безопасное расстояние. Если тупо с самой баржи - помашем ей ручкой. Если в барже сделать технологическую газоотводную дырень - ебейшие потоки пара и газа меняют плотность воды - и снова косплей Титаника.
Ну я бы не рассматривал обычную баржу вроде ОКИСЛЫ или баржу для песка. Скорее что-то вроде пикрела, конвертированную нефтяную платформу, полно таких.
>Считаем затраты времени и средств, что по итогу выгоднее. Идите нахуй, мыши, я стратег
Выгоднее не возвращаться с марса. Такие дела.
А если надо - тут как хочешь изъебнись, но ракета должна целехонькая.
А вертолеты и шары там не сработают, так что или учись пукать безопасно, что хз как сделать, или площадку умей готовить.
Возможно, илоний именно на это и расчитывает, чтоб котлованы не рыть и можно было залить бетониум и с него стартовать не повреждая ракету. Пока не вышло.
>Считаем, что выгоднее - держать вертодром или перебросить вертолеты из не столь отдаленных мест постоянного базирования.
То есть стартовый комплекс надо рядом с вертодромами строить чтоб можно было стрельнуть десяток вертолетов когда удобно?
>Керосин гидразин можно и откачать. Криогенику стравить экологически безопасно, если не ебанет не должно
Откачать куда? В шар или в вертолет? Ты помимо тросов еще и заправочную систему поднимать собрался теперь?
А кривогеника, ну да, может испарится до того как заморозит посевы и лишит аризонщину хлеба.
>Нинужны, можно и на бок класть,
Нельзя. Серьезно. Ты же хотел упрощать это все, а теперь добавляется сложность на поперечные нагрузки, добавляются посадочные системы чтоб ракета могла ложиться наземь не ломаясь.
>главное осторожненько и в какую нибудь старто-посадочную колыбельку из натяжных тросов, потому что >хрупкие дохуя
Ты на аэростате назад к стартово-посадочному комплексу не вернешься, его куда унесло, там и скрабаешь. А точнее - никак не скрабаешь, потому что ракета потеряна, а любые мероприятия для предотвращения этого настолько переусложняют систему, что проще собрать White Knight 3 и с него пускать уже.
>Короче дискасс вплотную приблизился к необходимости сопромат-экономических расчетов, с чем и поздравляю.
Он уже не раз был в этих окрестностях, это для меня дежавю.
>Пикрил - последствия хлопка с задымлением в воздухе, и на земле.
Опять же, Илоний, не там экономишь, площадку перестроить сильно проще чем ракету. Что сложно - это всю инфраструктуру, ваб, персонал и прочее запиливать, но оно для того и находится на расстоянии от стартовой площадки, что когда бадабум - их не распидорашивает.
А вот когда ты поднимешь заправленную ракету на шаре, а потом шар ракету на город уронит - тогда будет пиздос. Китайцы это даже без шара умудрились провернуть.
Аноним 13/08/23 Вск 09:48:15 760911 151
>>760861
>Новая модель предсказывает
Замечательно. Осталось эти предсказания проверить. Ведь предсказания той же лямбда-си-ди-эм успешно подтверждаются наблюдательными данными. Нахуя же пересматривать возраст Вселенной в сторону его увеличения?
>В 2021 году на основе этой модели возраст Вселенной был оценён в 13,797 миллиарда лет
Откуда это взяли-то? Не 13,797, а
>a measurement based on direct observations of an early state of the universe, which indicate an age of 13.787±0.020 billion years as interpreted with the Lambda-CDM concordance model as of 2021

>проливает новый свет на так называемую «проблему ранних галактик»
Бля, эта "проблема" решается корректировкой существующих моделей эволюции галактик. Пересмотр текущей космологической модели здесь не требуется - данные по галактикам никак не противоречат базовым современным представлениям о возрасте и эволюции Вселенной.
Короче, очередная претенциозная хуета с громким заголовком, призванная лишь привлечь внимание овощей. Ни о чем
Аноним 13/08/23 Вск 10:07:51 760912 152
>>760908
Ну вроде как мы сошлись на том, что 1) йоба-ракеты конечно, очень нежные, но 2) поднять их в воздух перед стартом, любым аппаратом, способным к зависанию - вполне решаемая задача. Несколько десятков автоматических замков для тросов, небольшое усиление корпуса в этих местах - вот и вся разница в конструкции ракеты. Насколько это экономически и по рискам оправдано по сравнению с классическим запуском - другой вопрос.

5к тонн - это миссия на Марс, как я понимаю? Даже Сатурн-5 весил всего 3к. Тогда больше вертолетов богу вертолетов, вертолет машина полезная, уже существующая и многофункциональная.

>>760910
>Прилетают вертолеты, цепляют кучу тросов
Стропальщики цепляют кучу тросов и радиально раскладывают их, вертолеты поднимают на безопасном расстоянии друг от друга. Возможно, не без помощи йоба-автопилотов, обученных на такую синхронную работу.

>А вертолеты и шары там не сработают
Вертолеты еще как сработают. Небольшой же летал уже, кажется. Там в разреженной атмосфере нужны бОльшие обороты ротора, в остальном так же всё.

>То есть стартовый комплекс надо рядом с вертодромами строить
В идеале да, но необязательно. Пригнать с расстояния 500-1000 км конечно дорого, но опять же, возможно - считаем что выгодней.

>Откачать куда? В шар или в вертолет?
Назад в цистерну. Наземную. По шлангу. Толстому. Кожаному. Предварительно спустив ракету низенько-низенько.

>заморозит посевы
Не преувеличивай. Пожароопасность процесса в связи с какой-нибудь статикой или курящим прапором - и то выше.

>Ты на аэростате
Да похуй на одноразовые аэростаты, действительно, ебатека получается, по здравому размышлению.


>площадку перестроить сильно проще чем ракету
>сложность на поперечные нагрузки
>Нельзя. Серьезно.

А вот тут очень обидно будет, если ты прав. Возможно, именно здесь собака порылась. Я в курсе, насколько эти йобы хрупкие и сделаны на пределе прочности для экономии массы.
Хотя можно сделать внешние усиливающие конструкции, принимающие и распределяющие нагрузки на изгиб, своего рода "корсет" для ракеты. И сбрасывать их вместе с тросами при старте.
Аноним 13/08/23 Вск 10:15:41 760913 153
image.png 78Кб, 225x225
225x225
>>760912
>Вертолеты еще как сработают. Небольшой же летал уже, кажется.
Аноним 13/08/23 Вск 10:20:22 760916 154
Аноним 13/08/23 Вск 10:23:43 760917 155
>>760910
> Китайцы это даже без шара умудрились провернуть.

История?
Аноним 13/08/23 Вск 10:36:58 760919 156
>>760912
>Даже Сатурн-5 весил всего 3к. Тогда больше вертолетов богу вертолетов, вертолет машина полезная, уже существующая и многофункциональная.
Хе-хе, что бы "поднять Сатурн 5" нужно примерно половина от всех Ми-26 построенных за всю историю.
Кстати, если уж говорить о
>И кстати в идеале это будет именно час, если не меньше - когда ракета готова к старту и должна быть поднята.
Типа "просто арендовать у частника"? По цене "летный час x количество вертолетов"? Ты действительно думаешь что тебе не придется вкладываться в свой персональный парк? По твоему у нас есть частники которые готовы дать в аренду даже несколько десятков тяжелых вертолетов одновременно в одном месте а операция "совместный подъем ракеты" не требует ни работы с вертолетами ни специальной подготовки пилотов?
Извини, но что бы поднимать Сатурн-5 тебе все таки придется заказать себе 150 вертолетов, нанять персонал, отгрохать всю инфраструктуру для них и т.д. И это все что бы сэкономить на газоводах, потому что вся остальная инфраструктура тебе все равно понадобится. Тебе все равно понадобится ебический МИК, потому что ракета сама себя не соберет, инфраструктура заправки, ибо топливо магически не материализуется, все диагностические системы и т.д. Таже транспортник потребуется, ибо ракета сама себя из МИКа не вывезет, а катающийся МИК дороже, чем катающийся транспорт для ракеты.
Аноним 13/08/23 Вск 11:05:38 760920 157
image.png 287Кб, 733x499
733x499
image.png 278Кб, 640x434
640x434
image.png 213Кб, 828x643
828x643
image.png 379Кб, 800x420
800x420
>>760912
>Стропальщики цепляют кучу тросов и радиально раскладывают их, вертолеты поднимают на безопасном расстоянии друг от друга. Возможно, не без помощи йоба-автопилотов, обученных на такую синхронную работу.
Решаемо, только где это, мы же про железку говорили.
>Вертолеты еще как сработают. Небольшой же летал уже, кажется. Там в разреженной атмосфере нужны бОльшие обороты ротора, в остальном так же всё.
Я думаю, ты понимаешь, почему этот пост >>760913
>В идеале да, но необязательно. Пригнать с расстояния 500-1000 км конечно дорого, но опять же, возможно - считаем что выгодней.
Логистический пиздец, надо прям эксклюзивные договоры ебашить чтоб тебе могли давать столько-то вертолетов с риском попасть под струю и в целом для опасных работ по перевозке взрывоопаски.
Это не говоря уже о том что надо штат пилотов обученных синхронно поднимать так. Такое вообще проворачивали? Синхронную перевозку кучей вертолетов?
>Назад в цистерну. Наземную. По шлангу. Толстому. Кожаному. Предварительно спустив ракету низенько-низенько.
Ага, михалыч задорно побежит под многотонную канистру с горючкой шланг цеплять, пока вертолеты бережно ее держат идеально на месте. Тут та же реакция уместна >>760913
>Не преувеличивай. Пожароопасность процесса в связи с какой-нибудь статикой или курящим прапором - и то выше.
Как бы то ни было, абы где горючку не сольешь.
На весу не сольешь.
Заправленную на землю нормально не положишь без лютейших изъебов.
Что-то пиздец усложнения пошли.
>Да похуй на одноразовые аэростаты, действительно, ебатека получается, по здравому размышлению.
Во, прогресс.
>А вот тут очень обидно будет, если ты прав. Возможно, именно здесь собака порылась. Я в курсе, насколько эти йобы хрупкие и сделаны на пределе прочности для экономии массы.
Атласы вообще загибались незаправленные, там топливо было частью несущей конструкции, лол. Я утрирую для красного словца, не надо воспринимать буквально.
>Хотя можно сделать внешние усиливающие конструкции, принимающие и распределяющие нагрузки на изгиб, своего рода "корсет" для ракеты. И сбрасывать их вместе с тросами при старте.
Мы приходим к тому, что воздушный старт звучит как пиздецкое переусложнение текущей реализации.
Напоминает как техбро пытаются регулярно переизобрести поезда: поды на рельсах, монорельсы, автопоезда, гиперзалупа и так далее. Когда как все это лишь по сути поезда с кучей дополнительных проблем и сильно дороже.
Так и тут: "просто поднять" не выйдет, когда вспоминаешь про то что еще ракете нужно, схема обрастает таким количеством усложнений и проблем, что ты просто должен задаться вопросом: а стоит ли оно того?
Аноним 13/08/23 Вск 11:18:01 760922 158
>>760919
Ну окей. Надеюсь, на очередной крупной миссии ехидный осколок бетона от дешевого и сердитого стартового стола не попадет в дюзу или бочину бака так, что похерит вообще всё, и очередной маршал будет тяжело утрачен.
А с другой стороны, все эти охуительные идеи от меня - просто мандраж в свете блистательного разъеба прототипа швитого старшипия, а на деле, возможно, все давно продумано, и риски минимальны с правильно спроектированным столом
Аноним 13/08/23 Вск 11:18:29 760923 159
>>760386 (OP)

А всё же обидно за Плутон. Тут десятую планету ищут, и им так "помогли".
Ну и похуй, что не в плоскости эклиптики. Может, Солнце ЗОХВАТИЛО Плутон от кого нить. Новые Территории, ёпт.
Аноним 13/08/23 Вск 11:35:02 760924 160
>>760922
>а на деле, возможно, все давно продумано, и риски минимальны с правильно спроектированным столом
This.
Проблемы стола для Старшипа были связаны с тем, что на нормальное отведение газов и борьбу с повреждением стола выхлопом просто забили. И решать их нужно (что и делается) доработкой стола, а не попыткой поднять Старшип при помощи 250 тяжелых вертолетов.
Аноним 13/08/23 Вск 11:36:13 760925 161
Смотрел в ТикТоке видосы и там попадались видео из космоса. Задумался, почему не видно звезд, полез в комменты и там многие это же спрашивают и ответили что-то про экспозицию камеры, что Солнце и Земля забирает весь свет на себя. Ну окей, ладно. А люди в открытом космосе тоже не видят звезд? И если, допустим, подальше от Земли отлететь, будут ли видны другие звезды через иллюминатор корабля?
Аноним 13/08/23 Вск 11:46:55 760927 162
image.png 2533Кб, 1920x1080
1920x1080
image.png 978Кб, 960x640
960x640
>>760922
>Ну окей. Надеюсь, на очередной крупной миссии ехидный осколок бетона от дешевого и сердитого стартового стола не попадет в дюзу или бочину бака так, что похерит вообще всё, и очередной маршал будет тяжело утрачен.
Они обычно и не попадают, Илон, на нормальных стартовых столах подобной хуиты не происходит. Просто построй нормальный стол с газоотводами.
>>760922
>мандраж в свете блистательного разъеба прототипа швитого старшипия, а на деле, возможно, все давно продумано, и риски минимальны с правильно спроектированным столом
"nj cgtwbfkmyj ,skj cltkfyj xnj,s ghjntcnbnm rfr dsit ujdjhbkjcm - djpvj;yjcnm cnfhnf c vfhcf ,tp djpvj;yjcnb gjcnhjqrb gjkyjwtyyjuj cnjkf/
Аноним 13/08/23 Вск 11:48:15 760928 163
>>760925
А люди на земле тоже не видят звезд? Ты себе такой вопрос задай.
Видят, когда темно и ничего не отсвечивает.
Аноним 13/08/23 Вск 12:00:42 760931 164
>>760928
Эм, в космосе тоже темно!
Аноним 13/08/23 Вск 15:04:47 760942 165
>>760899
Ракеты ещё древние китайцы проектировали, а самолёты - нихуя. Самолёты сложнее.
Аноним 13/08/23 Вск 15:09:01 760943 166
>>760942
Окей, но самолёты мне не интересны.
Аноним 13/08/23 Вск 15:48:27 760950 167
Почему спейсач мертв?
Аноним 13/08/23 Вск 15:53:18 760952 168
Почему просто не выроют пусковую шахту со дна марианской впадины до вершины эвереста и разгонять ракету на лифте в ней, а двигатели запускать только в самом конце? Бензина много сэкономим.
Аноним 13/08/23 Вск 16:26:49 760957 169
>>760952
> шахту со дна марианской впадины до вершины эвереста
Ну это был бы самый затратный гигапроект всего человечества, несопоставимый ни с чем до этого.
Аноним 13/08/23 Вск 16:53:47 760959 170
0в.jpg 50Кб, 1024x220
1024x220
>>760911
>>760865
Учёные были озадачены существованием звёзд, которые кажутся старше предполагаемого возраста нашей Вселенной. Таких, как Мафусаил — звезды-субгиганта, возраст которого по некоторым оценкам составляет 16 миллиардов лет. А также образованием галактик в ранней Вселенной, которые обнаружил телескоп Джеймса Уэбба. Это галактики, образовавшиеся всего через 300 миллионов лет после Большого Взрыва, при этом уровень эволюции и массы таких галактик обычно ассоциируется с миллиардами лет эволюции. Кроме того, это удивительно малые по размеру галактики, что добавляет ещё один элемент в загадку.

Раджендра Гупта, профессор физики Оттавского университета совместил модель Lambda-CDM и теорию усталого света Цвикки, переосмыслив красное смещение как гибридное явление. Теорию Цвикки Гупта дополнил эволюционирующими «константами связи», которе предположил Поль Дирак. Константы связи — это фундаментальные физические константы взаимодействия между частицами. Согласно Дираку, эти константы могли изменяться со временем. Если учитывать изменения констант, то время образования ранних галактик, наблюдаемых телескопом Уэбба, может быть увеличено с нескольких сотен миллионов лет до нескольких миллиардов лет. Это даёт объяснение аномалий масс и развития этих древних галактик.

Гупта предполагает, что интерпретация космологической постоянной — тёмной энергии, которая ответственна за ускоренное расширение Вселенной, — нуждается в пересмотре. Вместо этого он предложил интерпретацию эволюции констант связи. Такая модификация космологической модели помогает решить загадку небольших размеров и времени формирования галактик, наблюдаемых в ранней Вселенной. И, как следствие, предполагает, что возраст Вселенной 26,7 миллиарда лет, а не 13,7, как считается сейчас.
Аноним 13/08/23 Вск 16:58:26 760960 171
>>760959
Ну ладно, что тут. Пускай дорабатывают и проверяют, как гипотезу примут - изучу. А пока это не моя специализация и я заебался уже.
Было бы охуенно если бы разрулили эти ТЭ и ТМ при мне.
А вообще недавно мысль сверкнула - а что если эти теории струн и прочая, это какое-то нагромождение и усложнение, и создаст ложную модель которая работает на неверных предпосылках вроде постоянства констант? Приятно видеть, что рассматривают варианты с неконстантностью констант. А то некоторые шизы пердят что ученые свято верят в аксиомы.
Аноним 13/08/23 Вск 18:29:37 760969 172
001.jpg 14Кб, 640x294
640x294
002.jpg 32Кб, 450x426
450x426
>>760942
>Ракеты ещё древние китайцы проектировали, а самолёты - нихуя. Самолёты сложнее.
А самолёты проектировали древние египтяне, и по техническому уровню они примерно соответствуют древнекитайским ракетам. Пикрелейтед сделан из какого-то лёгкого дерева, и его даже в аэродинамической трубе продували. https://www.youtube.com/watch?v=zGVWcF4fssI

Можно даже приделать к древнеегипетскому планеру древнекитайскую ракету, и оно даже немножко полетит.
Аноним 13/08/23 Вск 18:37:59 760971 173
>>760969
>Можно даже приделать к древнеегипетскому планеру древнекитайскую ракету, и оно даже немножко полетит
Ага, полетит как орел на воздушном шаре
https://youtu.be/tgeyTIUybNM
Аноним 14/08/23 Пнд 00:05:40 760999 174
>>760911
> Бля, эта "проблема" решается корректировкой существующих моделей эволюции галактик. Пересмотр текущей космологической модели здесь не требуется - данные по галактикам никак не противоречат базовым современным представлениям о возрасте и эволюции Вселенной.
очередной охранитель будет обмазываться костылями, чем признает, что все чему его учили — хуета.
Аноним 14/08/23 Пнд 01:12:58 761001 175
>>760925
Звёзды видно, просто разница в яркости с освещенностью в районе земли от любого объекта очень большая. Короче как фонарем светить в ебало пытаясь разглядеть слабенький огонек вдали. А вот например в районе Плутона звёзды отчётливо видно уже даже на фоне плутона и его спутника, можешь с новых горизонтов посмотреть в базе фото, там полно звёзд. А так в космосе если нет вокруг освещенных солнцем объектов то при правильной настройке звезды будут на снимках. Собственно и с поверхности Луны можно получить, там атмосферы нет и она не засвечивает. Но если в кадре будет луна, то либо она будет пересечена, либо звёзд не видно.
Аноним 14/08/23 Пнд 10:20:03 761011 176
>>760959
>Учёные были озадачены существованием звёзд, которые кажутся старше предполагаемого возраста нашей Вселенной
Ну так это артефакт моделей звездной эволюции. Если пытаться оценивать возраст звезд "в лоб", то в некоторых случаях можно получить подобные абсурдные значения. Но если внести поправки с учетом расширения Вселенной, то все становится адекватно. Про этот Мафусаил несчастный часто любят пиздеть всякие альтернативно одаренные, но лишь от недалекого ума и элементарного невежества в данной теме
>>760999
>пук хрюк уииии
Аноним 14/08/23 Пнд 10:22:50 761012 177
>>760971
О, смотрите-ка, соевый научпокер в треде. Кого еще смотришь, соя? Трэшсмэша? Смузи пьешь?
Аноним 14/08/23 Пнд 11:34:28 761014 178
>>761012
> Смузи пьешь?
А если я смотрю научпоперов и пью пиво - норм?
Аноним 14/08/23 Пнд 12:05:59 761015 179
>>761014
Нет, нужно соответсовать образу. Срочно переходи на соевое смузи
Аноним 14/08/23 Пнд 14:12:51 761019 180
>>761015
Я пью смузи без сои, на обычном молоке.
Аноним 14/08/23 Пнд 15:33:30 761026 181
Почему радиоактивные элементы не распадаются сразу после возникновения, при взрыве звезд? При взрыве звезды же наверняка вокруг дохуилярды нейтронов, которые бомбардируют радиоактивные элементы.
Или распадаются и мы наблюдаем лишь оставшиеся крохи, которым невероятным чудом повезло?
Аноним 14/08/23 Пнд 15:39:08 761027 182
>>761019
Ну если пьешь смузи, то уже молодец. Образцовая соя. Волосы еще покрась и бороденку отпусти пидорскую
Аноним 14/08/23 Пнд 16:06:55 761030 183
Что во Вселенной больше нейтрино или фотонов?
Аноним 14/08/23 Пнд 16:27:20 761031 184
>>761027
Какая соя, дебил? Я же специально написал - без сои.
Аноним 14/08/23 Пнд 16:41:36 761032 185
Аноним 14/08/23 Пнд 17:53:21 761039 186
>>761031
Ты соя ебаная. Углеродный след свой устранил? На питание жуками перешел?
Аноним 14/08/23 Пнд 20:43:18 761050 187
16920282534501.jpg 58Кб, 512x512
512x512
Чё за кал.
Я бы им отдал свой телефон.
Аноним 14/08/23 Пнд 21:19:39 761053 188
>>760969
Кстати коровьи лепёхи очень похожи на тысячелетний сокол.
Аноним 14/08/23 Пнд 21:22:53 761054 189
>>760959
гупта из оттавы, блядь!!! Ёбаный индус из канады.... индусы все идут нахуй.
Аноним 14/08/23 Пнд 21:45:44 761055 190
>>761032
Хз. Что такое фотон?
Аноним 14/08/23 Пнд 21:51:13 761056 191
>>761026
>Или распадаются и мы наблюдаем лишь оставшиеся крохи, которым невероятным чудом повезло?
Это.
При бахе же дохера всяких говен высыпается, многие сразу распадаются, многие сколько-то лет.
Технецием, например, наверняка неслабо насрало, и он был бы на земляшке если бы ее собрать сразу после того катаклизма что создал то облако из которого она собралась.
Мы бы сразу померли от радиации.
А еще я подумал, что если и существует какое прям пиздец стабильное говно из острова стабильности, не этого где микросекунды, а подальше, лол, то оно тоже в тех звездах/взрывах/слияниясь нагенерилось и свалилось вглубь земляшки.
Мы найдем это говно в недрях астероидов и пересмотрим планетарные модели и модели ядерной физики.
Но это затравка для сайфая, на самом деле я не считаю что такой остров стабильности существует, распидорас сильно пересиливает ядро.
Аноним 14/08/23 Пнд 22:56:38 761059 192
>>761030
песчинок на пляже омаха
Аноним 14/08/23 Пнд 23:43:03 761061 193
>>761059
Это нечестный пример, песчинок на пляже омаха больше чем звезд в солнечной системе.
Аноним 15/08/23 Втр 20:17:31 761130 194
Фотон - это квант поля или квант излучения? Это ж разные вещи, не?))
Аноним 15/08/23 Втр 20:32:04 761135 195
>Учёный из Южной Кореи получил убедительные доказательства, что теории гравитации Ньютона и Эйнштейна могут быть ошибочными. На примере двойных звёздных систем он показал, что при слабых ускорениях звёзды движутся до 40 % быстрее, чем предсказывают теории Ньютона и Эйнштейна. Такое «экстремальное» поведение гравитации делает ненужными поиски тёмной материи и энергии — эти понятия становятся лишними. Похоже, мы на пороге революции в астрофизике и физике.
https://3dnews.ru/1091458/polucheno-ubeditelnoe-dokazatelstvo-alternativnoy-teorii-gravitatsii-ona-vihodit-za-ramki-teoriy-nyutona-i-eynshteyna
Ваше мнение
Аноним 15/08/23 Втр 20:47:46 761138 196
>>761135
Нет, не моё. Это мнение корейца. Пусть матан свой в рецензируемый журнал запостит, там и посмотрим.
Аноним 15/08/23 Втр 21:55:59 761149 197
Fluorescenceinc[...].jpg 531Кб, 2560x1707
2560x1707
PW-2012-08-10-p[...].jpg 35Кб, 700x538
700x538
hole.jpg 129Кб, 562x524
562x524
Event-Horizon-I[...].webp 342Кб, 3032x1706
3032x1706
Посоны, а мы ведь не знаем, что такое на самом деле является фотоном.
У него нет инерциа́льная система отсчёта.

Попытка рассмотреть систему отсчета, движущуюся со скоростью света, приводит к математическим и физическим парадоксам. Если бы мы могли смотреть на мир из "системы отсчета фотона", пространство и время вокруг нас бы обнаружили странные и неинтуитивные особенности. Например, интервалы времени становились бы бесконечно длинными, а расстояния становились бы бесконечно короткими.
Аноним 15/08/23 Втр 23:02:34 761158 198
>>761130
>Фотон - это квант поля или квант излучения?
Подожди, то есть "фотона-частицы" не существует?
Аноним 15/08/23 Втр 23:04:34 761159 199
>>761149
>а мы ведь не знаем, что такое на самом деле является фотоном.
За себя говори!
Аноним 15/08/23 Втр 23:24:32 761162 200
>>761149
>бесконечно короткими
Это как, блять, так?
Аноним 16/08/23 Срд 07:15:21 761172 201
>>761149
А не все ли равно насчет СО фотона? Он переносчик взаимодействия а не вещество, для него времени не существует.
Аноним 16/08/23 Срд 09:14:35 761176 202
>>761172
Фотон при определенных условиях вполне может обвестись массой. И у него появится вполне собственное время.
Брат фотона Z W бозовы тоже являются переносчиком взаимодействия, но этом время для них очень важно.
Аноним 16/08/23 Срд 10:10:16 761178 203
>>761130
>Это ж разные вещи, не?
Не, это одно и то же. Фотон - квант электромагнитного поля=квант электромагнитного излучения
>>761149
>а мы ведь не знаем, что такое на самом деле является фотоном
Ну если ты не знаешь нихуя, то это твои проблемы. Современной науке же это известно. Об этом сказано в посте выше. Фотон - квант электромагнитного излучения/поля. Грубо говоря - частица света
Аноним 16/08/23 Срд 14:08:38 761189 204
>>761158
>Подожди, то есть "фотона-частицы" не существует?
Смотря что называть "частицей".
Аноним 16/08/23 Срд 14:14:00 761190 205
>>761176
>Фотон при определенных условиях вполне может обвестись массой.
При каких?
Аноним 16/08/23 Срд 14:36:25 761191 206
>>761178
>Не, это одно и то же. Фотон - квант электромагнитного поля=квант электромагнитного излучения
Глупость) Поле и излучение - не одно и то же. Поле - это среда, в которой распространяется излучение. Ну смотри, например, на примере звуковой волны. Вот у тебя есть вода (поле), в ней распространяется звук (излучение). Квант поля - молекула воды. А квант звука? Тоже молекула воды? А если звук теперь распространяется в воздухе - то теперь квант звука - молекула воздуха? Вот ты дурак, блять))
Так же и со светом. Квант электромагнитного поля - кубик. А квант излучения - фотон. Но фотон не может быть кубиком. Или может?
Аноним 16/08/23 Срд 16:41:10 761197 207
>>761190
При высоких напряженностей полей, не обаятельно электромагнитных.
Внезапно в веществе.
Особый пример это сверхпроводники и ферромагнетики. В них фотон наделяется массой сходным образом с механизмом хиггса.
Аноним 16/08/23 Срд 18:38:13 761199 208
>>761197
Не. Если у фотона появляется масса - это больше не фотон. Не?
Аноним 16/08/23 Срд 19:05:53 761200 209
>>761199
С хуяли?
Фотон по определению это квант эм поля. В общем смысле это как скачком меняет 4-потенциал в определенной окрестности пространства-времени. То что есть масса или нет влияет на него суть.
Аноним 16/08/23 Срд 19:16:55 761201 210
Как относитесь к теории Горькавого о том что тёмная материя - это скопления чёрных дыр переживших не одну сотню циклов сжатия-расширения?
Аноним 16/08/23 Срд 19:23:47 761202 211
>>761200
>Фотон по определению это квант эм поля.
Не поля, а излучения.
Аноним 16/08/23 Срд 19:58:12 761205 212
>>761201
Блядь, и кто только наркоманов в насу принимает? Я конешно не наркоман, а просто алкаш, но я хотел бы в насу попасть. У меня теорий до ебени матери про вселенную. Они все с формулами и картинками. Чё я хуже этого горькавово?
Аноним 16/08/23 Срд 20:03:02 761206 213
Будет ли на самом деле колонизировать марс в ближайшие 20 лет? Этож тупо тратить триллионы долларов в течении овердохуя десятелитей, пока марс наконец-то не сможет какую-то прибыль приносить или хотя бы самостоятельно существовать
Аноним 16/08/23 Срд 20:17:19 761207 214
>>761205
>Чё я хуже этого горькавово?
В журналах не печатаешься
Аноним 16/08/23 Срд 21:53:45 761211 215
А в друг темная материя это гравитация "протекающая" из параллельных вселенных?
Аноним 16/08/23 Срд 21:58:47 761212 216
>>761211
А каков механизм протекания?
Аноним 16/08/23 Срд 22:18:55 761213 217
>>761212
Унитазно-смывной механизм
Аноним 16/08/23 Срд 22:29:46 761214 218
>>761213
Это невозможно, так как в таком случае тёмная материя состояла бы из воды и говна и легко бы обнаруживалась.
Аноним 16/08/23 Срд 22:36:55 761215 219
>>761206
Колонизировать - не, но базу можно построить.
Аноним 16/08/23 Срд 22:47:33 761216 220
>>761215
Отправят людей без возврата или будут ждать технологию, чтоб можно было посадить ракету и затем улететь? Эта технология нескоро, наверное, появится, а в планах уже даже это десятилетье людей отправить. Ктож согласится на возможно навсегда отправиться на марс и жить в консервной банке всю жизнь, вкалывая как ишак?
Аноним 16/08/23 Срд 22:59:42 761217 221
>>761216
Чтоб нормально летать на Марс, надо запилить базу и индустрию на Луне. С нее очень удобно осуществлять безракетные запуски хоть на само Солнце. Топливо для кораблей нужно только для торможения и возвращения назад. Впрочем на Марсе можно заправляться метаном на месте.
Аноним 16/08/23 Срд 23:17:37 761218 222
>>761214
Говно в космосе не обнаружить, нюхательных телескопов пока не изобрели, а вода и углеводороды с метанами в рассеянном виде летают в космосе. У меня есть формулы и картинки, есличо.
Аноним 16/08/23 Срд 23:19:18 761219 223
>>761217
безракетные запуски с луны, ага. А залупоньку не хочешь?
Аноним 16/08/23 Срд 23:28:49 761220 224
>>761217
>надо запилить базу и индустрию на Луне
Это само по себе очень нескоро. Значит про колонию/базу на марсе в ближайшие 20 лет это пиздабольство?
Аноним 16/08/23 Срд 23:43:30 761222 225
>>761219
Дебил, ты обосрался, спок!
При 50g для разгона до лунной Второй космической (2,4 км/с) нужна катапульта всего 6 км длиной, время разгона 5 сек.
Буквально 2 рельсы по 6 км, проложенные рядом друг с другом. А поскольку ускорение такое маленькое, а не как в корабельных рейлганах по 10000 g, прослужит катапульта очень долго. Что касается людей, то их можно запускать с ускорением 8g на 12км-катапульте: на низкую орбиту и с первой космической (и с небольшой пороховой пшикалкой для апогейного пшика для циркуляризации орбиты).
Аноним 16/08/23 Срд 23:53:49 761226 226
>>761222
Обосерешься ты ж\д в 6 км прокладывать в говённом лунном грунте, плюс тяжеленный упор для начала разгона... да какого хуя, это всё равно ракета на рельсах получается. Чё ты мне зачехляешь?
Аноним 17/08/23 Чтв 00:24:36 761227 227
>>761222
А как же центрифуга? Есть проект, где таким образом пуляют с Земли болванки на почти орбиту, на Луне такое точно прокатит
Аноним 17/08/23 Чтв 00:37:02 761228 228
>>761220
Как пойдет. Текущий уровень технологий вполне позволяет лунные базы с производством на месте. Проблема больше в первом шаге. Его очень легко зайфейлить и поэтому подкидывают соломки.
Но про Марс близлежащие 20 лет можно и не мечтать.
Аноним 17/08/23 Чтв 01:28:21 761229 229
>>761228
>Текущий уровень технологий вполне позволяет лунные базы с производством на месте.
Для начала надо убедиться что лёд есть не глубже пары метров
Аноним 17/08/23 Чтв 11:30:00 761261 230
>>761229
На фиг тебе лед? Хочешь на Луне джакузи с шлюхами?
Даже если лед на глубине десяток метров, копать на Луне значительно проще чем на Земле. Посадки планируются все равно планируют не в базальтовых морях.
Аноним 17/08/23 Чтв 11:44:48 761265 231
>>761228
Сколько воздуха с собой на луну брать?
Аноним 17/08/23 Чтв 11:45:37 761266 232
>>761261
>На фиг тебе лед?
Тупой вопрос.
Аноним 17/08/23 Чтв 11:56:07 761267 233
>>761265
На самом деле не много.
Главное тут система утилизации углекислого газа и аммиака, которой уже нужно много энергии.
Аноним 17/08/23 Чтв 12:23:51 761268 234
>>761267
Немного - это сколько? 100 кг? Тонну? Регенераторы надо брать?
Аноним 17/08/23 Чтв 12:48:01 761273 235
>>761268
Установка собственно, она не большая около 60 кг. Больше весит силовая установка для нее. Для регенерации воздуха для одного человека занятый физической работой нужно около 8 киловатт.
Всякие регенераторные шашки естественно надо брать как НЗ на случай аварий.
Впрочем на Луне один хрен будут плавить реголит, а нем почти половина по массе это кислород. Так же около процента в нем натрий, который будет служит буффером для углекислого газа.
Аноним 17/08/23 Чтв 19:10:39 761327 236
>>761273
А сам реголит будет без кислорода плавиться?
Аноним 17/08/23 Чтв 19:58:37 761340 237
Почему американцы перестали летать на луну?
Аноним 17/08/23 Чтв 20:08:56 761342 238
Почему на луне нельзя ориентироваться с помощью компаса?
Аноним 17/08/23 Чтв 20:13:24 761346 239
>>761342
На Луне нельзя использовать обычные магнитные компасы для ориентирования из-за отсутствия у неё значительного магнитного поля, которое есть, например, у Земли. Это связано с отсутствием у Луны глобального жидкого железного ядра, которое создавало бы магнитное поле, подобное земному.

На Земле компас работает из-за взаимодействия магнитной стрелки компаса с магнитным полем Земли. Стрелка компаса выравнивается в направлении север-юг в магнитном поле Земли.

Так как у Луны нет атмосферы и магнитного поля, компас на Луне не будет указывать на север, как это происходит на Земле. Для ориентирования на Луне чаще используют астрономические ориентиры, такие как положение Солнца, Земли и других небесных тел.

chatgpt
Аноним 17/08/23 Чтв 20:56:37 761355 240
>>761327
Вполне будет.
Это же ебаный пироксен + включения металлического железо-никель. Для них не нужны большие температуры. А кислород тем более не нужен.
Аноним 18/08/23 Птн 18:37:40 761417 241
Анон, а гравитационные волны они быстрее скорости света "перемещаются" в пространстве? Или один хуй свет и гравитационные волны с какого-то события прилетят на Землю одновременно?
Аноним 18/08/23 Птн 19:33:11 761424 242
>>761417
>а гравитационные волны они быстрее скорости света "перемещаются" в пространстве?
Нет.
Аноним 19/08/23 Суб 01:07:35 761451 243
>>761417
скорость света придумали шизики, которым больше делать нехуй
Аноним 19/08/23 Суб 09:52:21 761471 244
Аноним 19/08/23 Суб 15:30:19 761484 245
Возможно ли война между существами расположеными в разных звездосистемах, не нарушая законы физики?
Как такое будет протекать?
Гравитационные окопы и кварк-глюонные пирамидки?
Аноним 19/08/23 Суб 15:45:45 761485 246
>>761202
Эфир Хиггса. Свет проходит насквозь.
Аноним 19/08/23 Суб 15:50:15 761486 247
А может ли свет двигаться если нет пространства-времени? Сам по себе? Нет пространства. Нет Времени. А он хуячит сам. Без всякой вселенной. Или ему что то нужно для спонсирования существования?

А что будет если кинуть песок например (какое то вещество) за пределы вселенной, где нет материи и пространства и времени, писок удариться о край мира или вылетит за пространство и перестанет существовать?
Аноним 19/08/23 Суб 16:20:32 761488 248
Стикер 63Кб, 436x435
436x435
>>761484
>Как такое будет протекать?
Долго.
Центурианцы запиливают массдрайвер, долго и точно считают, и запуливают большущую болванку аккурат в сраную земляшку. Болванка прилетает через 30 лет и благодаря низкому либидо прилетает незамеченной аккурат до того момента пока не въебывается в луну потому что забыли про луну.
Аноним 19/08/23 Суб 16:20:58 761489 249
>>761486
Не, не может. Это как загрузить программу в память компа без памяти.
Аноним 19/08/23 Суб 16:38:40 761490 250
>>761488
То есть они сломают нам приходы и отходы?
Да за такое можно и гравитационной волной шлепнуть.
Аноним 19/08/23 Суб 18:10:02 761500 251
>>761486
>А что будет если кинуть песок например (какое то вещество) за пределы вселенной
У Вселенной нет пределов
Аноним 19/08/23 Суб 20:42:37 761619 252
Что будет если засунуть хуй в черную дыру?
Аноним 19/08/23 Суб 20:48:08 761625 253
1692467287600.png 370Кб, 2000x2555
2000x2555
Аноним 19/08/23 Суб 21:59:18 761680 254
Аноним 19/08/23 Суб 22:19:27 761683 255
>>761619
Лучше пощекоти горизонт хуйцом, но не засовывай. Он спагеттифицируется и вырастет.
Аноним 19/08/23 Суб 22:22:54 761685 256
>>761683
Насколько далеко самая близкая черная дыра? Спрашиваю для друга.
Аноним 19/08/23 Суб 22:26:10 761689 257
Аноним 19/08/23 Суб 22:30:21 761692 258
Как же всё заебало нахуй. Сколько ещё надо циклов чтобы понять этот ебучий космос наконец? Надо блядь наролить себе каким-то хуем в какой-то пизде айкю 200, потом выжить, потом не сойти с ума, потом выучиться нахуй как-то, всё это переосознать и высрать конечную теорию всего сущего в единой формуле. И вот только тогда всё снова схлопнется, расширится и мы идём снова из обезьян нахуй.
Аноним 19/08/23 Суб 22:43:15 761697 259
>>761689
А может ли быть поближе, но не замеченная? И можно ли будет обнаружить в ближайшие годы пока собираю звездолет
Аноним 19/08/23 Суб 22:45:42 761699 260
image.png 91Кб, 300x168
300x168
>>761692
Смотри PBS Spacetime. Я не стал понимать космос, даже наоборот, стал лучше понимать насколько мало я понимаю, и мое понимание моего непонимания стало расти, показывая что я знаю еще меньше чем мог бы.
Но просмотр этих передач с ОДаудом почему-то не вызывает фрустрацию несмотря на упомянутый эффект, а доставляет.
Аноним 19/08/23 Суб 22:56:58 761709 261
>>761689
Какова вероятность, что рядом с Солнечной системой(хотя бы 10 парсек) есть Бродящая Черная Дыра?
Аноним 19/08/23 Суб 22:57:27 761710 262
>>761699
>почему-то не вызывает фрустрацию несмотря на упомянутый эффект, а доставляет
Как так?
Аноним 19/08/23 Суб 23:03:56 761714 263
image.png 146Кб, 360x270
360x270
>>761710
Не знаю. Мэтт добрый такой, как-то не создает ощущения что я дебил, наоборот, несмотря на огромный путь в познаниях, все увеличивающийся передо мной, мне приятно что предстоит проходить его вместе с ним.
Аноним 19/08/23 Суб 23:49:29 761736 264
>>761714
Это безумие, понимаешь? С каким бы лицом не рассказывали, но как кино это не воспринимается, потому что это всё про то где мы есть и с этим ничего не сделать.
Аноним 20/08/23 Вск 01:11:56 761750 265
>>761451
> скорость света придумали шизики, которым больше делать нехуй
Ну мало ли. Может там природный аналог пузыря Алькубьерре, ведь гравитационные волны тоже по сути корячат пространства. Я поэтому и спросил.
Аноним 20/08/23 Вск 14:48:55 762266 266
Аноним 20/08/23 Вск 14:57:22 762271 267
изображение.png 54Кб, 449x295
449x295
изображение.png 79Кб, 453x426
453x426
>>762266
Связь с Луной-25 прервалась. Похоже, что АМС разбилась.
Аноним 20/08/23 Вск 21:34:50 762476 268
>>761355
Охуел ты, металлург космический.
Аноним 20/08/23 Вск 21:39:39 762477 269
А вот сингуляторность, из которой большой взрыфв въебал, она же по сути чёрная дыра невъебенной массы, а чёрные дыры же не взрываются, только джетами попукивают. Как она въебала то?
Аноним 20/08/23 Вск 21:45:44 762481 270
Аноним 21/08/23 Пнд 10:38:14 762578 271
Где группа телеги
Аноним 21/08/23 Пнд 11:58:05 762590 272
Что будет если разогнать монетку до скорости света и врезать в планету? Пройдет ли монета насквозь, через едро и т.д.?
Аноним 21/08/23 Пнд 11:58:33 762591 273
Аноним 21/08/23 Пнд 12:52:14 762603 274
>>762590
Если ты имеешь ввиду разогнать до релятивистских скоростей, то нет не пробет.
Вообще любые тела влетающие в планету на больших скоростях глубже километра за роются, вместо этого они при ударе тупо исправятся вместе с грунтом, оставляя только фрагменты, которые успели отколупиться и плавно затормозиться в атмосфере.
Если ты сраную монетку разгонишь до релятивистских скоростей, то она начнет вступать в ядерные реакции еще в атмосфере, вызывая почти ядерный врыв, без какой либо эффектной пенетрации в темные глубины планеты.
Аноним 21/08/23 Пнд 13:27:05 762629 275
>>761619

Его оторвет приливной силой еще на подлете.
Аноним 21/08/23 Пнд 18:23:11 762719 276
image.png 1697Кб, 1332x888
1332x888
Не знаю, где еще спросить. В /sci/ какой-то трэш происходит. Что случится с АЭС в случае какого-то сценария вроде скоропостижного зомби-апокалипсиса? Если некому будет поддерживать их работу или хотя бы просто безопасно вырубить. Ждет ли планету в таком случае превращение в одну большую зону отчуждения спустя какое-то время?
Аноним 21/08/23 Пнд 18:29:06 762724 277
>>762719
большинство сколь-нибудь современных реакторов относительно безопасно отрубятся сами. Дальше - по обстоятельствам, но более вероятен сценарий Три Майл Айленда, а не Фукусимы с Череобылем, т.е. расплав активной зоны, реактору пизда, но срань в целом локализована.
>Ждет ли планету в таком случае превращение в одну большую зону отчуждения спустя какое-то время?
нет, АЭС-ок очень мало.
Аноним 21/08/23 Пнд 21:03:29 762759 278
>>762719
В случае сценария, при котором некому будет поддерживать работу АЭС, существует риск серьезных ядерных аварий. Реакторы и хранилища отработанного ядерного топлива требуют постоянного охлаждения и мониторинга.

Краткосрочные последствия:
Если охлаждение прекратится, ядерные реакторы могут перегреться и, в худшем случае, произойти расплавление активной зоны. Это может привести к выбросам радиоактивных материалов.

Среднесрочные последствия:
Даже если реакторы автоматически остановятся, системы охлаждения отработанного ядерного топлива всё равно требуют электроэнергии и обслуживания. В противном случае может произойти утечка радиоактивных материалов.

Долгосрочные последствия:
Если аварии произойдут на нескольких станциях, это может привести к образованию больших зон радиоактивного заражения, схожих с Чернобыльской Зоной Отчуждения.

В итоге, если не предпринять меры по обеспечению безопасности АЭС, такой сценарий может привести к экологической катастрофе на большой территории. Планета не превратится в "одну большую зону отчуждения", но локальные зоны сильного радиоактивного заражения станут непригодными для жизни на долгое время.
Аноним 21/08/23 Пнд 21:12:46 762762 279
>>762590
В реальном мире невозможно разогнать монетку до скорости света, так как это противоречит законам физики — масса монетки станет бесконечно большой, и потребуется бесконечное количество энергии для её ускорения.

Однако, допустим, монетка двигается с очень большой, но субсветовой скоростью. В этом случае энергия столкновения будет чрезвычайно высокой. Это приведёт к огромному выбросу энергии, аналогичному взрыву, и может вызвать глобальные катастрофы на планете: от мощных землетрясений и цунами до изменения орбиты планеты. Монетка не пройдёт насквозь, она и планета взаимодействуют в форме экстремального высвобождения энергии.
Аноним 22/08/23 Втр 02:03:10 762788 280
>>761619
Зависит от размеров/плотности дыры, есть мнение что в СМЧД можно провалиться без спагеттификации хуя, так и останешься со своими 9см
Аноним 23/08/23 Срд 12:07:48 763072 281
Нужен ли скафандр в космосе, если у нас некий участок пространства стопроцентно без комической радиации и любых, не то что микро, но даже нано, метеоритов?
Типа, скафанд, фактически, нужен только для головы (чтобы дышать и говорить в микрофон) и рук (чтобы трогать и не обмораживать руки), а например очко жопы прикрывать нет смысла. Разница между давлением на берегу реки на пляже (1 атм) и на маленькой глубине (2 атм) всего 1 атмосфера. Разница между вакуумом (0 атм) и воздухом на Земле или в шаттле (1 атм) - 1 атмосфера. Если под водой при переводе давлений в 1 атм мне вода в жопу под напором не заливается, то и при выходе голышом в открытый космос, по идее, очко поддувать из сопла даже минимальную тягу не должно
Аноним 23/08/23 Срд 13:12:14 763092 282
>>763072
Нужен обязательно. Проблема в нулевой влажности. Вся вода с твоей кожи моментально испарится заодно охладив то, что останется.
Надо объяснять почему это плохо или нет?
Аноним 23/08/23 Срд 13:33:56 763101 283
>>763092
>Вся вода с твоей кожи моментально испарится заодно охладив то, что останется.
Выхожу в космос с насухо вытертой кожей. Проблемы?
Да и изжопная влага образует ледяную корочку, из-под которой вода уже не испаряется, совсем как со всеми водоледяными спутниками всех газгигантов или дальних планет
Аноним 23/08/23 Срд 14:21:18 763110 284
>>763072
>Разница между давлением на берегу реки на пляже (1 атм) и на маленькой глубине (2 атм) всего 1 атмосфера. Разница между вакуумом (0 атм) и воздухом на Земле или в шаттле (1 атм) - 1 атмосфера.
Проблема в том, что тело лучше приспособлено удерживать давление, направленное внутрь. Для отрицательного перепада условий на Земле никогда не было, поэтому эволюция к такому не подготовилась. Мгновенного пиздеца не будет, но для начала такой астронавт жидко пёрнет. Потом по обстоятельствам.
Аноним 23/08/23 Срд 15:09:46 763130 285
>>760386 (OP)
Слышал, что в галактике что у центра, что на краю одинаковая скорость.

О какой скорости идёт речь? Линейной?
Аноним 23/08/23 Срд 15:53:31 763173 286
изображение.png 74Кб, 1364x623
1364x623
Аноним 23/08/23 Срд 17:14:26 763223 287
>>763130
Средняя скорость или почти температура, если бы звезды были как частицы газа.
Есть такая величина как вириал.
Из нее можно выразить среднею скорость элементов для самогравитирующих систем зная гравитационный потенциал и наоборот.
В случае галактики обычно берутся звездные скопление и считается их средняя скорость по широте спектральных линий.
Другой метод по эмиссионному излучению газа, подсеченным каким-либо источником.
Если еще метод по запрещенным линиям нейтрального водорода.
Аноним 23/08/23 Срд 18:42:12 763245 288
Вот прямо сейчас берём и наш Юпитер заменяем железокаменной планетой такой же массы, что и Юпитер. Только вместо водорода и гелия планета состоит из того же, из чего Меркурий, Венера, Земля и Марс.
Что измениться в солнечной системе? Что станет с орбитами лун Юпитера? Какого объёма будет новый Юпитер?
Аноним 23/08/23 Срд 19:41:50 763267 289
Что будет, если поместить маленькую черную дыру массой 10 масс Земли и радиусом чуть больше пары метров за Плутоном? Она будет влиять как-то на орбиту Земли?
Аноним 23/08/23 Срд 20:08:04 763273 290
>>763072
Когда-нибудь делали тебе засос? Нет? Ожидаемо. Ну ты сам себе сделай. А теперь представь что то же самое, раз в десять сильнее и по всему телу. Будет НЕКОМФОРТНО, мягко говоря. Нужен костюм поддерживающий давление.
Аноним 23/08/23 Срд 20:20:21 763278 291
>>763245
>Что измениться в солнечной системе?
Практически ничего.
>Что станет с орбитами лун Юпитера?
Станут меньше тормозиться о частицы несуществующей атмосферы улетающей с планеты, но и меньше ускоряться от огромного альбедо. Оба фактора заметны только на астрономических временных отрезках.
Короче: ничего не изменится в их орбитах.
>Какого объёма будет новый Юпитер?
379,600,000,000,000,000,000,000 кубометров.
Радиус - 44 тысячи километров.
Аноним 23/08/23 Срд 22:27:40 763296 292
>>763101
Алё, там ещё внутриклеточная вода. Мне, конечно, стоило написать "из кожи", но я и так думал, что понятно будет.
Итак, у тебя полностью высохнет кожа. Вероятно ещё кровь закипит, там в плазме тоже дохуя воды, но у меня не достаточно компетенции, чтобы это утверждать.
Аноним 23/08/23 Срд 22:39:35 763297 293
Можно ли сделать аппарат для газового гиганта или Венеры, который не будет приземляться на поверхность где его раздавит, а включит двигатель и зависнет на какое то время в атмосфере, где будет собирать данные об атмосфере и делать снимки?
Ну или оседлать на какое то время ветра Юпитера, Нептуна, где под действием ветра он не погрузится глубже. Этакий воздушный змей из легких сплавов.
Почему космические аппараты пролетают мимо Урана и Нептуна. Почему на них обиты не выводят что-то?
Можно ли сделать телескоп вроде JW и поместить его на орбиту Юпитера или Сатурна. Сможет ли он делать такие же хорошие и четкие снимки пока вращается по орбите?
Аноним 23/08/23 Срд 23:14:51 763304 294
>>763296
Ничего не вскипит, не пизди. Кожа повредится от вакуума, но не от "вскипания".
Аноним 23/08/23 Срд 23:33:24 763306 295
>>763297
>Можно ли сделать аппарат для газового гиганта или Венеры, который не будет приземляться на поверхность где его раздавит, а включит двигатель и зависнет на какое то время в атмосфере, где будет собирать данные об атмосфере и делать снимки?
Можно, только профита не очень. А если брать двигатель он должен быть безтопливный, а такие только при сильно низких давлениях работают.
Друга проблема это торможение. Аппарату на Юпитере тяжело сбросить орбитальную скорость и его разбивает не давление, а сверхзвуковая ударная волна.
>Почему космические аппараты пролетают мимо Урана и Нептуна. Почему на них обиты не выводят что-то?
До сих пыхтеть десятки лет, тем более с выходом на орбиту. Разработка столько же. Да еще нужен целый штат людей где-то брать, которые свою жизнь на это положат. Можно когда-нибудь в далеком будущем.
>Можно ли сделать телескоп вроде JW и поместить его на орбиту Юпитера или Сатурна. Сможет ли он делать такие же хорошие и четкие снимки пока вращается по орбите?
Может, но это бессмысленная трата ресурсов.
Аноним 24/08/23 Чтв 00:28:31 763311 296
>>763304
Ты говно почему без хлеба ешь? Причину и следствие понимаешь? Вакум - причина, вскипание - следствие. Не то "вскипание", которое ты в чайнике видишь. Оно больше похоже на кавитацию, но без сильной динамики, это охуеть сильное разрежение. Среды разные. Вскипанием это называется из-за похожести процесса образования пузырёв при кипении воды, ёпта. Только причины абсолютно разные. Избыток давления и нихуя давления.
Аноним 24/08/23 Чтв 01:29:06 763315 297
>>763072
Очевидно разница между нулем и единицей не тоже самое что между единицей и двойкой. В случае единицы и двойки давление есть и если резко не менять его то тебе будет пахуям очень долго что при одной что при двух и даже при сотне атмосфер. А вот ноль атмосфер, это буквально означает что вокруг тебя нет ничего что создавало бы давление, т.е. полная пустота и поскольку внутри тебя оно есть, ты будешь стремиться наполнить эту пустоту собой, пока все твои клетки не спустят давление до нуля.
Аноним 24/08/23 Чтв 05:01:03 763320 298
>>763072
>Разница между давлением на берегу реки на пляже (1 атм) и на маленькой глубине (2 атм) всего 1 атмосфера. Разница между вакуумом (0 атм) и воздухом на Земле или в шаттле (1 атм) - 1 атмосфера. Если под водой при переводе давлений в 1 атм мне вода в жопу под напором не заливается, то и при выходе голышом в открытый космос, по идее, очко поддувать из сопла даже минимальную тягу не должно
В большинстве физических явлений, где фигурирует перепад давления, присутствует множитель вида "P2/P1". И когда P1 стремится к нулю - этот множитель стремится к бесконечности.

Так что вакуум - это стрёмная хуйня. Жидкости испаряются сами по себе, скорость истечения газов соизмерима с тепловой скоростью молекул, КПД тепловых машин на таком перепаде стремится к 1, растворимость газа в жидкости стремится к нулю и.т.п. Биологическим тканям от градиента давлений с нулём становится очень хуёво.
Аноним 24/08/23 Чтв 05:10:35 763321 299
>>763304
>>763311
Там ещё гораздо раньше вскипания крови начнутся явления, аналогичные декомпрессионной болезни у аквалангистов. Из всех тканей полезет растворенный в воде азот, распидорашивая клеточные стенки и вызывая тромбозы с эмболией. И полезет газ значительно эффективнее, чем при резком всплытии с 30м, по причине вышеописанной. 1/0.001 это значительно больше, чем 30/1. Там и недоусвоенный кислород полезет.
Аноним 24/08/23 Чтв 06:59:38 763324 300
Как выглядит гиперкуб вписанный в гипершар?

Можно ли выбраться из трёхмерного пространства в внутри пузыря гипершара в трёхмерное пространство снаружи гипершара не дожидается, пока лопнет плёнка гипершара, отделяющая внешнее трёхмерное пространство от внутреннего?
Аноним 24/08/23 Чтв 08:16:49 763329 301
Почему поверхность магнетара такая идеально ровная?
Аноним 24/08/23 Чтв 08:27:09 763330 302
>>763321
Подожди, не нагружай его так, пусть сначала азы усвоит, а потом уже подробности. Кстати, я думаю вакум ещё и говно из жопы высосет, даже сфинктер не поможет.
Аноним 24/08/23 Чтв 08:44:48 763339 303
>>763329
С хуя ли она ровная?
>>763324
Так внутри гипершара вроде же 4х пространство. И в 3х пространстве он существовать не может, точно так же как шар не может объективно быть в 2х, те на плоскости. Плоскость, прямая и точка - эти разбиения придуманы для упрощения. А всё что выше 3х - это уже фантазии. Может в каких-то расчётах это и помогает, но их реальность пиздец как сомнительна, даже я бы сказал, все эти гипертесеракты - всё это хуйня и выдумки.
Аноним 24/08/23 Чтв 09:16:14 763342 304
>>763339
> С хуя ли она ровная?
ОНА НЕ ПРОСТО РОВНАЯ, ОНА ПОЧТИ ИДЕАЛЬНАЯ НАХУ
Аноним 24/08/23 Чтв 09:21:13 763343 305
>>763342
Откуда ты это придумал?
Аноним 24/08/23 Чтв 13:31:51 763368 306
>>763320
Вот тоже об этом хотел сказать, это вообще особенность математики, вроде кажется ну что такое ноль, ерунда же. А начинаешь делить просто единицу на все большее число и чтобы достичь нуля у тебя делитель улетает на бесконечность. Причем с другими числами такого нет, они просто гиперболу тебе отодвинут, но не как с нулем потребуется полет в бесконечность.
Аноним 24/08/23 Чтв 18:29:09 763383 307
>>763329
Быть неровной ей там ТЯЖЕЛОВАТО
Аноним 24/08/23 Чтв 18:30:44 763384 308
>>763330
>>763321
Мало ли что вы думаете себе там, есть исследования по последствиям организмам в вакууме, ничего никуда не вылезает кроме газов из открытых полостей (кишечник и легкие).
Аноним 24/08/23 Чтв 19:11:53 763389 309
>>763278
>Радиус - 44 тысячи километров.
В смысле? Это же всего лишь на треть меньше радиуса Сатурна
Аноним 24/08/23 Чтв 19:17:18 763390 310
>>763330
>Подожди, не нагружай его так, пусть сначала азы усвоит, а потом уже подробности. Кстати, я думаю вакум ещё и говно из жопы высосет, даже сфинктер не поможет.
Порно-ужастик ВАКУУМ-АНИЛИНГУС
Аноним 24/08/23 Чтв 19:24:59 763392 311
>>763389
В прямом.
Юпитер ОСНЕ ЖИРНЫЙ. Массу возьми в тоннах, подели на пять - получишь количество кубометров, потом по формуле объема шара получишь радиус.
Аноним 24/08/23 Чтв 20:30:23 763396 312
regnumpicture14[...].jpg 897Кб, 1920x960
1920x960
Good-times-as-n[...].jpg 92Кб, 696x365
696x365
Почему космические станции выглядит как несколько коридоров-залуп? Что мешает запустить в космос большие просторные блоки помещения, чтоб в будущем нарастить большое здание в космосе?
Аноним 24/08/23 Чтв 20:45:08 763397 313
>>763396
Очевидно потому что размер того, что ты запускаешь в космос ограничен размером ракеты. Самое большое помещение, которое в космосе было, было на Скайлэбе, а это по сути переделанная верхняя ступень лунного сверхтяжа.
Аноним 24/08/23 Чтв 20:45:41 763398 314
>>763390
>>763330
А одном из шаттлов из-за сбоя системы сброса астронавт жопой прилип к унитазу, но ничего страшного не произошло.
Аноним 24/08/23 Чтв 20:49:18 763399 315
>>763396
В ракету не влезат, сверхтяжи порезали чтоб большие модули пулять.
Мусковский грунтодёр когда ещё полетит.
Аноним 24/08/23 Чтв 20:57:28 763400 316
>>763398
>А одном из шаттлов из-за сбоя системы сброса астронавт жопой прилип к унитазу, но ничего страшного не произошло.
Звучит как мемный порно-заголовок про Смешариков, где Копатыч застрял в деревенском толчке
Аноним 24/08/23 Чтв 21:00:54 763401 317
>>763396
>Почему космические станции выглядит как несколько коридоров-залуп? Что мешает запустить в космос большие просторные блоки помещения, чтоб в будущем нарастить большое здание в космосе?
Диаметр обтекателя мешает. Огромный дом к ракете не привяжешь, а если и привязать - охуеешь от атмосферного сопротивления.

Впрочем, этот лимит можно немножко поднаебать надувными модулями. Где-то так в 3-5 раз диаметр можно увеличить.
Аноним 24/08/23 Чтв 21:01:44 763402 318
>>763396
>Почему космические станции выглядит как несколько коридоров-залуп? Что мешает запустить в космос большие просторные блоки помещения, чтоб в будущем нарастить большое здание в космосе?
Диаметр обтекателя мешает. Огромный дом к ракете не привяжешь, а если и привязать - охуеешь от атмосферного сопротивления.

Впрочем, этот лимит можно немножко поднаебать надувными модулями. Где-то так в 3-5 раз диаметр можно увеличить.
Аноним 24/08/23 Чтв 21:07:27 763403 319
Скажите, а может ли быть при каких нибудь реальных условиях комбинации вариантов звезды, вариантов орбиты планеты, вариантов состава планеты и атмосферы планеты, и вращения планеты так, чтобы дневная тень на планете была светлее, чем дневной свет? Чтобы днём свет у поверхности был если не лучами тьмы, то лучами тени, а тень (то есть непопадание солнечного света), оказывалось бы более освещённым?
Мне почему-то кажется, что это крайне сложно организовать, но возможно и что ключ к решению данной задачи в химиелюминисценции, реагирующей на УФ/свет/тепло-комбинацию местного солнца, которое заставляет темнеть поверхность под прямыми солнечными лучами и использовать солнечный свет как катализатор для светополглощения и/или разложения с поглощением тепла и света.
Но дальше этого мне как-то не думается и придумывается
Аноним 24/08/23 Чтв 21:16:34 763405 320
>>763403
Невозможно. То что освещает тень и светлую часть будет освещать, освещенная часть будет суммарно светлее.
Аноним 24/08/23 Чтв 22:05:50 763411 321
Как может быть вселенная 93 миллиардов лет размером, если ей всего 14 миллиардов лет? Типа большой взрыв произошел не в одной точке, а в нескольких сразу что ли? Если он произошел в одной точке, то дольше 1 скорости света все эти протоны, нейроны не могут двигаться, а следовательно они не смогли бы дольше 14 миллиардов лет улететь от нас, как тогда утверждают, что вселенная 93 миллиардов лет размером?
При большом взрыве, сразу же говорят создались некоторые большие взрывы, а не должны были бы они все сразу поглотить, пока не успело все улететь от точки большого взрыва?
Аноним 24/08/23 Чтв 22:06:41 763412 322
>>763411
*некоторые черные дыры
Аноним 24/08/23 Чтв 22:40:05 763416 323
>>763411
>Как может быть вселенная 93 миллиардов лет размером, если ей всего 14 миллиардов лет?
Вселенная расширяется быстрее скорости света.

> Типа большой взрыв произошел не в одной точке, а в нескольких сразу что ли?
Он произошл во всех точках сразу, по сути это не "взрыв" из какой-то точки, а просто начало расширения во все стороны.
Аноним 24/08/23 Чтв 23:00:54 763421 324
>>763384
Так газики из жидкостей вылезут и тушку раздует к хуям. Пердежи полезут из анальной жопы, толкая фекальные массы, как поршень, а так как анус очка всётаки сжат, то станет своего рода дроссельной шайбой. Вопщем говно как из пульверизатора будет брызгами хуярить, а возможно вобще в виде аэрозоля.
Аноним 25/08/23 Птн 05:11:33 763454 325
>>763421
>Пердежи полезут из анальной жопы, толкая фекальные массы, как поршень, а так как анус очка всётаки сжат, то станет своего рода дроссельной шайбой.
Бля, а ты человек? Не пердел никогда, только гипотетически рассуждаешь? Или всегда дристопоносишь, и твердого стула никогда не имел?
Потому что, черт побери, запросто можно пердеть не обсираясь.
Аноним 25/08/23 Птн 06:15:10 763460 326
>>763411
Вселенная появилась до большого взрыва. Большой взрыв -- это когда она начала расширяться с огромной скоростью.
Никто не знает размеров вселенной, нам известен только размер наблюдаемой её части -- это тот кусок, границы которого улетали от нас со скоростью света в тот момент, когда космос стал проницаем для фотонов.
Далёкие звезды от нас удаляются не за счёт своей скорости, а за счёт нарождающегося пространства. Это снимает в некотором смысле ограничение на движение со скоростью выше скорости света.
Аноним 25/08/23 Птн 08:32:33 763466 327
>>763454
Когда снаружи лютое разрежение - полезет с говном 100%.
Аноним 25/08/23 Птн 08:40:25 763469 328
>>763460
На границе наблюдаемой вселенной в данный момент такие же ведь галактики как наша. То есть там нету этой параши, которую телескопы видят, они видят устаревшие данные. То есть ещё молоденькие скопления, молодые звёзды, тоесть этих блядских астрономов-вуайеристов можно вполне сажать за подглядывание за несовершеннолетними. А что виел бы наблюдатель, находясь в галактике на границе нашей наблюдаемой вселенной? Наверняка ведь то же самое, что и мы. Так что говорить о возрасте вселяшки, основываясь только на том наскока далеко мы видим - безрассудно, так ведь?
Аноним 25/08/23 Птн 12:11:04 763487 329
>>763469
Уже все, не молоденькие, слишком много звёзд, слишком много углерода насинтезировалось. В общем что-то не так в теориях и моделях, либо вселенная старше, либо свет стареет как минимум
25/08/23 Птн 18:34:38 763531 330
Понаблюдав за звездным небом, я слегка прихуел от количества летающих точек. В безоблачую погоду ночью их хорошо видно. Выглядит как небольшая звезда, пересекает видимую часть неба за 2 минуты примерно. Направления разные. За 15 минут, я насчитал 7 объектов. Спутники Маска? Но они вроде строем летают. Что это?
Аноним 25/08/23 Птн 18:51:17 763536 331
>>763531
Строем они не летают. Это только пока они не вышли на свои орбиты.
Аноним 25/08/23 Птн 19:10:27 763539 332
>>763531
Просто рандомные спутники скорей всего. В хорошую ночку я тоже кучу вижу.
Батя астроблядством не увлекается и считает что это над нашими краями специально летают т.к. особые объекты рядом. Донести до него без визуальных пособий рандомность орбит не удалось.
Бомбардировку Луны отработали - хохлам приготовиться Аноним 25/08/23 Птн 20:15:27 763544 333
❗️Путин поручил Роскосмосу и АСИ к декабрю рассмотреть вопрос о создании космических аппаратов для предельно низких орбит — до 200 км, сообщили в Кремле.
Аноним 25/08/23 Птн 22:28:40 763551 334
>>763544
Типа, новая орбитальная станция МИР?
Аноним 26/08/23 Суб 00:52:53 763572 335
Кто из бегунов быстрее всего добежал из города А каждый в свой город, при условии, что нам неизвестно место нахождения ни одного города?
Аноним 26/08/23 Суб 03:53:50 763591 336
Извините, что задаю наитупейший вопрос, но: откуда брать снимки планет/звёзд и прочих космических тел и явлений? желательно в достойном качестве и чтобы ресурс регулярно пополнялся новыми фотокарточками
Аноним 26/08/23 Суб 09:20:26 763602 337
>>763572
>Кто из бегунов быстрее всего добежал из города А каждый в свой город, при условии, что нам неизвестно место нахождения ни одного города?
Чему равен n в равнении y=xq+((56w/40j)7s/8n3x)+5f?
Аноним 26/08/23 Суб 09:22:55 763603 338
>>763531
>Что это?
Большинство - спутники/дроны/китайские фонарики. Но есть среди них и настоящие НЛО
Аноним 26/08/23 Суб 12:44:24 763637 339
>>763602
Откуда уравнение?
Аноним 26/08/23 Суб 15:40:43 763653 340
>>763637
>Откуда уравнение?
Из головы
Аноним 26/08/23 Суб 19:38:34 763678 341
image.png 676Кб, 960x556
960x556
image.png 1730Кб, 1280x853
1280x853
image.png 1108Кб, 827x601
827x601
image.png 1672Кб, 1200x800
1200x800
Хочу написать твёрдый сайфай.
Помоги с описание параметров в системе двух тел + мелкого мусора.
Дано: газовый гигант едва выходящий за пределы зоны жидкой воды зоны комфорта. ГГ этот размерами навскидочку как какУран, ну или чуть больше. У него есть большой близкий спутник Марсоид. Настолько близко, насколько возможно, чтобы не падать вниз на какУран. При этом магнитные поля какУрана по Марсоиду не особо-то и проходятся, потому что юбка магнитного поля и радиационные пояса этого ГГ начинаются парой с десятков тыщ км по внешнюю сторону Марсоида. Там есть и другие луны, но, как и у Сатурна, мелкие и газопылевые или ледокаменные, а вот большой спутник Марсоид у какУрана один, как Ганимед у Сатурна. Тем не менее, спутники за Марсоидом пролетают к Марсоидом достаточно близко и сами достаточно массивные, чтобы приливнозахваченный Марсоид при пролёте близжайшего спутника на грудус-другой проворачивается, а значит, что каждые Х поворотов вокруг какУрана, Марсоид таки делает один полноценный оборот вокруг своей оси. Это, в свою очередь поддерживает какую-никакую тектонику и магнитное поле самого Марсоида, что, вкупе с внешним поле какУрана, даёт этому Марсоиду полноценную электромагнитку для защиты от радиации, и дополнительную теплоту от уже не так уж и греющего далёкого солнца.
Собствнно, учитывая что Марсоид лёгкий в сравнении с реальными Землёй и Венерой, надо ему поддать атмосферы поболее 1, а значит пусть как на сатурнианском Титане - 1,5атм.
Жидкая вода, атмосфера и т.д. Все это факторы в совокупности оказываются достаточными для приемлемой по Земным меркам жизни. И это фантастическое допущение. Достаточная - не значит, что климат тот, или местность приемлемая или что жизнь там вообще будет но это уже суть сюжета, а не суть вопроса.
Так вот, а теперь вопрос: как подвести твердоту к обоснованию заданных на данных момент условиях, чтобы было твёрдо и чётко, со вкусом твердоты, но без послевкусия космической фентези на губах и космооперы на вкусовых сосочках читателя?
Какое будет небо на Марсоиде?
Велик ли в небе какУран?
Раз планета меньше, то и время другого толка измерения, так?
Раз в сколько часов/дней Марсоид будет сжимать булки, пока луна поменьше игриво щекочет гравиатцией марсоидынй анус?
Если все параметры орбиты привязаны к массе и скорости, то как часто на Марсоиде будет совершён один месяц, т.е. оборот вокруг какУрана? И раз в сколько - оборот вокруг своей оси (т.е. день будет длинее месяца, что логично)?
Нужно ли херачить много маленьких лун-шох за Марсоидом для прокручивания или вообще кольца, чтобы они своей массо равномерно влияли, или одной-двух более больших как раз и нужно ставить в обрбитальный резонанас? Если да, то как часто будут встречаться Марсоид и внешние планетоиды?
Так же понимаю, что соразмерность Марс-Уран для планеты и её спутника какая-то нереалистичная как у Земли с Луной, поэтому, если что считаем пару какУрана и Марсоида двойной планетой, а камушки вокруг - спутниками системы Марсоид-какУран, точно как Плутон-Харона и спутников. А какие максимально нередкие и наиболее вероятные варианты образования такой планетарной системы могут быть? Rogue Planet притянутая и удачно уложившаяся в именно такую уютную орбиту для поддержания воды, а атмсоферы, биосферы и жизни - ну это вообще пиздец маловероятно. Нановероятно, я бы даже сказал. Это вариант слишком фентези, а нужен сайфайный.
Будут ли какие-то характерные, логические погодные или гипотетические биосферные особенности проявления мира этого Марсоида в заданных условиях? Мне кажется, раз в Х часов при пролёте сзади луны поменьше, по экватору Марсоида будут идти бури и ураганы с ветрами. Но почему? Кроме как "примерно почувствовал" не могу объяснить. И ещё, читал теорию, что у приливнозахваченной планеты без спутника на вечной светлой стороне должен быть вечный гигашторм из пыли. Тут такое может быть ли, надо ли?
Вообще, размеры тел - Марсоид и какУран норм ли взял, или их стоит подкорректировать?
Аноним 26/08/23 Суб 19:39:54 763681 342
>>763678
> как Ганимед у Сатурна.
Как ТИТАН
Самофикс
Аноним 26/08/23 Суб 21:58:24 763690 343
>>763678
Ну давай разберем по частям, тобою написанное ))
Если планета гигант вроде Урана запихнуть в зону обитаемости, то у него сильно разбухнет атмосфера. Чем-то он будет смахивать на Венеру - горячая плотная атмосфера, только из метана, аммиака и воды. В таких условиях вода сможет переходить в сверхкритическое состояние, отчего вся планета будет покрыта специфическим облаками с очень высоким альбедо. Рядом с ней она будет ярко светить почти как звезда.
Из-за раздутости атмосферы в ней будут бушевать очень мощные штормы, которые будут дотягиваться до "ледяной" мантии, тем самым наводя сильное глобальное магнитное поле мультипольного характера. Радиационные пояса в таких пиздец ебанный, почти как у Юпитера.
Прилив на твоем Марсе это ебанный пиздец. Почти как Аду, вулканы будут пердать лаву аж на орбиту.
Аноним 27/08/23 Вск 09:53:34 763716 344
>>763690
>Если планета гигант вроде Урана запихнуть в зону обитаемости, то у него сильно разбухнет атмосфера
Но она не находится в этой зоне. Она выходит за пределы, но лежит близко. Допустим, у нас есть 100м радиусом кольцо - зона комфорта. А этот какУран хлдит и круги наворачивает по радиусу 102-105-108м. Крорче, близко, но во вне.
>Рядом с ней она будет ярко светить почти как звезда.
Кто она? Рядом с кем?>>763690
>Из-за раздутости атмосферы в ней будут бушевать очень мощные штормы, которые будут дотягиваться до "ледяной" мантии, тем самым наводя сильное глобальное магнитное поле мультипольного характера.
А как это связано?
>Радиационные пояса в таких пиздец ебанный, почти как у Юпитера.
Плохо. А можно ли получить кольцо-дели в поясах магнитных полей, как можно получить дыры в кольцах Сатурна пуская туда спутников-пастухов? Ну то есть, в теории возможно ли избавиться от проблемы радиации какУрана? Или, наоборот, врубить собственное динамо Марсоида, чтобы оно успешно сопротивлялось магнитке какУрана?
>Прилив на твоем Марсе это ебанный пиздец. Почти как Аду, вулканы будут пердать лаву аж на орбиту.
Бля... А без этого никак? Варианты исправления ситуации возможны? Чтобы ебеших вулканов не было или чтобы они были не постоянными и не еьешими?
Аноним 27/08/23 Вск 11:56:53 763721 345
>>763716
Ураны/непутны(ледяные гиганты) начинают раздуваться на уровне орбиты Юпитера, что довольно далеко от зоны обитаемости.

Атмосфера у гигантская тепловая машина, на ледяных гигантах и так бушуют лютые штормы, не смотря на то, что получают очень мало тепла от Солнца. А много тепла от Солнца запускает глобальную суперротацию атмосферы. Поскольку у ледяных гигантов нет твердой поверхности, то атмосфера плавно переходит в сверхкритическую жидкость, а еще ниже в суперионный лед. Конвекция от атмосферы и до ядра собственно и генерирует магнитное поле. Если запускается суперротация, то масштабы генерации магнитного поля усиливаются.
В зависимости характера конкреции, магнитное поле генерируется не дипольное, а минимум квадрупольное. Хотя не ясно как усиление конвекции поливает на характер магнитного поля.

Да, никак.
Приливные силы зависят от размера тела, чем больше тело, тем сильнее приливные силы как таковые и обширнее они действуют. Так Луна создает сильнее прилив для Земли, чем наоборот, при этом она значительно легче Земли. Марс довольно крупное тело и прилив будет соответствующих.
Либо отводи из зоны приливного захвата, либо сувай тело поменьше.
Аноним 27/08/23 Вск 15:59:27 763732 346
>>763721
>Либо отводи из зоны приливного захвата, либо сувай тело поменьше.
Если отводить, то насколько далеко?
Если поменьше, то насколько?
Аноним 27/08/23 Вск 19:48:35 763749 347
>>761061
Лол, три раза прочитал прежде чем понял прикол....
Аноним 27/08/23 Вск 19:50:05 763750 348
>>761050
Хм.
Земля, даже с Луны, выглядит как апельсин...
Аноним 27/08/23 Вск 19:56:23 763751 349
>>761201
Черная дыра - это область за горизонтом событий
С ### ли ей расширяться - сужаться, да еще и циклично?
Это по Фрейду, что ли?
Аноним 27/08/23 Вск 19:59:25 763752 350
>>761355
А ты умный. Два чая тебе.
Аноним 27/08/23 Вск 20:01:09 763753 351
>>761211
Что и к чему притягивает эта твоя "протекакающая" гравитация?
Аноним 27/08/23 Вск 20:03:07 763754 352
>>761340
С разморозкой.
Уже летают снова. Мало того, еще и Луна теперь - не конечная цель, а лишь местный космовокзал, что ли
Аноним 27/08/23 Вск 21:58:09 763757 353
Аноним 27/08/23 Вск 22:05:18 763759 354
>>763732
Если брать массу и размер Марса, то надо уводить далеко за миллион километров, может даже больше.Это на треть больше чем орбита Луны вокруг Земли. Это не уменьшить вулканизм как таковой, но зато не будет ебаных лавовых фонтанов.
Если хочешь уменьшать размер спутника, то он должен быть примерно с Луну, но в таком случае масса будет маленькая и соответственно не способна удерживать атмосферу.

Есть еще другая проблема. Крупные спутники вокруг теплого ледяного гиганта будет напрочь лишен атмосферы. На этапе формирования такой системы атмосфера спутника перетечет на планету или вообще спутник упадет на планету. Короче твоя хотелка естественным путем не может образоваться.
Аноним 27/08/23 Вск 22:16:15 763760 355
>>763759
>Есть еще другая проблема. Крупные спутники вокруг теплого ледяного гиганта будет напрочь лишен атмосферы. На этапе формирования такой системы атмосфера спутника перетечет на планету или вообще спутник упадет на планету. Короче твоя хотелка естественным путем не может образоваться.
Звучит хуёво. Даже хуёвее, чем кажется. Если только не создать какую-то идею, при атмосфера с какУрана затекает на и утекает с Марсоида. Но тогда, получается, Марсоид должен быть вообще пиздец близко к "поверхности" какУрана, а это значит, быть частью атмосферы и уже конечно пересекать предел Роша.
Аноним 27/08/23 Вск 22:58:34 763761 356
Разговор про то что юпитер мог бы быть второй звездой натолкнул на мысль.

Есть газопылевое массой, скажем, 10 солнечных.
Как понять, получится ли из него одна звезда массой 10 солнечных и куча планет, или две звезды разных масс. Или вообще большая часть массы не будет участвовать?
Когда и как может возникнуть такая ситуация когда газовый гигант реально может быть второй звездой в системе, или когда планеты образуются уже все, поздно?
Аноним 27/08/23 Вск 23:35:41 763762 357
>>763267
Не больше, чем маленькая черная дыра твоей мамаши
Аноним 27/08/23 Вск 23:42:32 763763 358
>>763324
Если пленка тоже гипер, то как бы очевидно, что нет.
Если пленка существует в меньшем числе измерений, (меньшем , чем 4, - например, как квадрат на кубе), - то очевидно, что да.
Реквестирую фото гипершарика. И гиперПятачка, и гиперОслика ИаИа
Аноним 27/08/23 Вск 23:46:30 763764 359
>>763324
Или ты о том, что гипершар можно интерпретировать как внешнее пространство, и внутреннее?
(грубо говоря, куб можно интерпретировать как квадрат, и "внешних" пять стенок)
Тогда - не знаю, мне и самому интересно.
Реквестирую капчу в виде гипершара
Аноним 27/08/23 Вск 23:53:38 763765 360
>>763761
Ничего из такого облака не получится. Облако должно быть большим, чтобы была движуха, чтобы звездообразование шло постоянно, гиганты быстро взрывались обратно, мелочь выбрасывалась, и всё это способствовало созданию в облаке областей с разной плотностью.
Аноним 27/08/23 Вск 23:54:33 763766 361
>>763591
Фоткай на свой пылесос
Аноним 27/08/23 Вск 23:54:56 763767 362
>>763591
Фоткай на свой пылесос
Аноним 27/08/23 Вск 23:56:39 763768 363
>>763678
Переименуй какУран на кукАрен, а Марсоид на Срамоид
Аноним 27/08/23 Вск 23:58:16 763769 364
>>763768
На дваче хуйни не посоветуют
Аноним 28/08/23 Пнд 00:04:54 763770 365
Аноним 28/08/23 Пнд 00:11:36 763771 366
>>763403
Солнышко стает черной дырой, и не светит.
А луна как светила, так и светит.
(А аккреционный диск идет нахуй)
Аноним 28/08/23 Пнд 00:17:48 763772 367
>>763403
Умные растения (или солнечные батареи) включаются, (днем), и поглощают свет (а ночью опять светлеют, от скуки).
Переход (из одного состояния в другое) может быть мгновенным).

Вариаант 3: на черную-черную землю вечно сыпет белый-белый снег, который мгновенно тает на дневном свете.... А здесь должны были быть стихи эмо-кошко-девушки
Аноним 28/08/23 Пнд 01:13:40 763774 368
кринжанул с говнописяки >>763403
Аноним 28/08/23 Пнд 03:16:25 763777 369
>>763403
Изначально вопрос лишён смысла и туп. Может ещё пусть нигры станут светлее белых людей?
Аноним 28/08/23 Пнд 03:20:49 763778 370
>>763751
Пиздец ты ссыкун набожный - на двоче очковать написать слово "хуй"... Пшёл отседа, говно филологическое, тебя лев толстой в аду в жопу выебет
Аноним 28/08/23 Пнд 16:00:27 763808 371
>>763777
Не имею ничего против хорошенькой негриТянки в беленьких платьице, шляпке и босоножках. Знаете, сколько есть разных оттенков белого?
Это, лев ты толстой, тред тупых вопросов, а не тред тупых ответов.... однако....
Аноним 28/08/23 Пнд 16:46:48 763809 372
>>763808
Зелёный - это тоже оттенок белого в своём роде. Против хороших годных нигритянок я против ничего и никогда не имел, а вовсе даже котировал и ебал как мог.
А то што ты меня левом толстым назвал, это конечно лестно, но я тебя ибать в аду не хочу. Потому что, есличо - я в рай попаду. Так что извини, анон.
А правда, почему ты не пишешь слово "хуй"? Нахуя эти решоточки ставишь? Мне вот реалли интэрэсно. Может ты сектант?
28/08/23 Пнд 19:14:53 763824 373
Screenshot 2023[...].png 115Кб, 255x374
255x374
>>763809
не, он не из этих…🤣
Аноним 28/08/23 Пнд 19:34:21 763832 374
>>763751
>С ### ли ей расширяться - сужаться, да еще и циклично?
Не ей, а Вселенной до размеров в сотни световых лет, что будет вполне достаточно для того чтобы дыры не слиплись в одну.
Аноним 28/08/23 Пнд 19:38:51 763833 375
>>763602
10-20, нэ болшэ, ара. Можэт 21, но эта толко если в срэду с утро.
28/08/23 Пнд 23:35:01 763863 376
>>763572
А был ли город?
Была ли планета?
Может всё это только у тебя в VR-е вычисляется и давно ты спишь с начала времён?

Коммутация не всегда связывает только две точки A и B…
не так всё просто, как сперва может показаться?
28/08/23 Пнд 23:35:49 763864 377
>>763833
Ара может! Ара может…
28/08/23 Пнд 23:50:05 763867 378
>>763751

Так ведь они же реактивными струями излучения Хокинга отталкиваются друг от друга и, хитро искажая пространство-время, быстро путешествуют, в противоположных направлениях, за горизонты совместных событий?

Фотоны, как носители информации, никогда не покидают окрестностей чд в пределах радиуса Шварцшильда?
29/08/23 Втр 00:10:14 763868 379
>>761149

Не «бесконечно короткими», а предельно малыми, если помножать их на сверхмалую массу фотона?

Однако, если, измерять время фотонами, при/в каких условиях оно может изменять скорость своей «длительности» и относительно чего?
29/08/23 Втр 00:14:44 763869 380
>>763867
Как вы можете об этом спрашивать или утверждать, ведь вы даже никогда этого не наблюдали?
Аноним 29/08/23 Втр 00:20:36 763870 381
>>760821
> Гравитация - это геометрический эффект искривления
Это чисто твои маняфантазии. Уровня интерпретаций квантовой механики
Аноним 29/08/23 Втр 00:34:30 763873 382
Почему некоторые металлы притягиваются магнитом, а некоторые - нет? Почему в достаточно сильном магнитном поле даже лягушки притягиваются?
Аноним 29/08/23 Втр 02:08:13 763880 383
>>763870
Гравитация это свойств эфира. Возможно даже что источником гравитации является нейтрино али другая частица. Эфир пронизан этими частицами во всех направлениях, как броуновское движение. Тела частично экранируют эти чястицы, поэтому они приятягиваются. Так как нейтрино с неэкранируемой стороны прилетает больше, и они толкают тела ро направлению друг к другу.

Это примудал один выживший из ума шиз (не я). И его теория наиболее красивая из существующих.
Аноним 29/08/23 Втр 03:45:14 763884 384
>>763880
хуйня этот кефир
Аноним 29/08/23 Втр 07:19:26 763887 385
image.png 499Кб, 1024x767
1024x767
image.png 293Кб, 1024x767
1024x767
image.png 1145Кб, 880x880
880x880
image.png 599Кб, 700x930
700x930
>>763763
>Реквестирую фото гипершарика
>>763764
>Реквестирую капчу в виде гипершара
Аноним 29/08/23 Втр 10:10:28 763895 386
Аноним 29/08/23 Втр 12:08:08 763906 387
>>763887
А как будет выглядеть сфера в комплексном двухмерном пространстве?
А как эта же сфера будет выглядит при ее проектирование на взаимносопряженные изотропные вектора?
Аноним 29/08/23 Втр 12:20:27 763908 388
>>763906
>А как будет выглядеть сфера в комплексном двухмерном пространстве?
Как два круга, соединяющихся всего одной точкой. Типа цифры 8
Аноним 29/08/23 Втр 13:27:18 763914 389
1693287587135.png 13Кб, 64x148
64x148
Аноним 29/08/23 Втр 13:46:26 763915 390
Застревает ли еда и вода в горле в космосе из-за отсутствия гравитации?
Аноним 29/08/23 Втр 14:04:01 763917 391
>>763915
Сколь могут пальцем упихивают, а дальше перистальтика работает.
Аноним 29/08/23 Втр 15:18:33 763921 392
15504083105582.jpg 586Кб, 1600x1061
1600x1061
>>760386 (OP)
Что скажете, аноны-астраномы? Пиздят индуские приборы или реально на луне такая ЖАРА? И почему такая жара стоит? И непонятно из сообщения, это жара стоит постоянно или максимально в определенные часы, и только на полюсе. И почему только на полюсе? И почему раньше не замечали таких высоких температур?
https://tass.ru/kosmos/18598465
>Как сообщил информационному агентству Press Trust of India сотрудник ISRO Би Дарукеша, полученные данные оказались неожиданными для специалистов. "Мы все считали, что температура на поверхности может составлять от 20 до 30 градусов по Цельсию, но она составляет 70 градусов по Цельсию. Это на удивление выше, чем мы ожидали", - цитирует агентство слова ученого
Аноним 29/08/23 Втр 20:19:32 763944 393
>>763921
В аризонщине асфальт тоже до стольки накаляется, а там день/ночь побыстрее будут.
Аноним 29/08/23 Втр 20:25:01 763945 394
>>763921
Походу таки реально жарко. Данные подоспели по спектральному составу реголита и там оказалось много серы и вообще не типичный состав.
Вероятно всего реголит там слабее рассеивает свет, отчего Солнце может нагревать более глубокие слои, а охлаждаются только самые верхние слои. Получается этакий парниковый эффект только не в газе, а в пыли.
Предлагаю, что там ебнулась комета, отчего сильно окислила породы, а поскольку это полярный регион, то минералы слабо подвержены выветриванию и восстановлению.
Аноним 29/08/23 Втр 22:26:51 763946 395
>>763945
>>763944
Ну тогда получается, что на Марсе тоже высокие температуры, нет?
Ну кроме полюсов. Хотя на Луне почему то температура оказалась высокая 70 градусов. Сколько же тогда в районе лунного экватора?
Все равно нихуя не понимаю, американцы гуляли по Луне и не замечали какая высокая температура вокруг. И что раньше никто не додумался замерить температуру? А если задумался, то какого хрена температура в 70 градусов вызывает удивление?
Я вообще думал, что на Луне всегда ниже нуля. Или около нуля.
Аноним 29/08/23 Втр 22:53:38 763947 396
>>763946
температура поверхности и температура атмосферы разные вещи, дебс. американцы не босиком гуляли, а в толстенных ботинках скафандра.
Аноним 29/08/23 Втр 23:17:55 763949 397
>>763946
С хуя ли на Луне должна быть постоянная низкая температура, или какая-либо постоянная температура? Луна на таком же удалении от Солнца, что и Земля, значит средняя температура будет приблизительно такой же (чуть ниже из-за отсутствия парникового эффекта). При этом атмосферы нет, а день и ночь по полмесяца, поэтому очевидно, что дневная сторона будет очень горячей, а ночная очень холодной.

>американцы гуляли по Луне и не замечали какая высокая температура вокруг
Американцы высаживались на Луну всегда утром и не задерживались до пекла. Как это будет с долгосрочными миссиями Артемиды хз.
Аноним 29/08/23 Втр 23:20:37 763950 398
>>763946
>Ну тогда получается, что на Марсе тоже высокие температуры, нет?
Нет, не получается. Луна на той же орбите что и Земля. Марс - дальше.
>американцы гуляли по Луне и не замечали какая высокая температура вокруг
Они на рассвете высаживались, реголит остывший после ночи был.
>А если задумался, то какого хрена температура в 70 градусов вызывает удивление?
Не знаю какого хрена температура в 70 градусов вызывает у тебя удивление. Как я говорил - асфальт в аризонщине до стольки же прокаляется, а у реголита альбедо как у асфальта.
>Я вообще думал, что на Луне всегда ниже нуля.
С Плутоном путаешь. Или с Минмусом
Аноним 30/08/23 Срд 00:47:02 763951 399
>>763950
>Они на рассвете высаживались, реголит остывший после ночи был.
И че? Это как в анекдоте полетите на Солнце. А не сгорим? Ты руковоство партии за дураков не считай, ночью полетите.
>>763949
>Американцы высаживались на Луну всегда утром и не задерживались до пекла.
Так вот. Американцы там тусовались сутками, я про следующие высоки на луну.
>Ну тогда получается, что на Марсе тоже высокие температуры, нет?
Нет, не получается. Луна на той же орбите что и Земля. Марс - дальше.
Вообще, не важно кто на какой орбите. Это слабо зависит от удаленности. Другие факторы играют еще большую роль.
У спутников Юпитера еще большие разрывы между собой по температуре, а находятся они тоже на одной орбите, ну почти.
>температура поверхности и температура атмосферы разные вещи, дебс
Во-первых, дебс это ты, какая нахуй атмосфера у Луны. Тысячные доли от Марсианской?
Да это понятно. Ну и какая должно быть темпера у поверности пол-тора метра например? Если температура 70 градусов?
Градусов 35 то будет? А ты такой в скафандре прыгаешь, а под скафандром у тебя еще одежда.
И да не отрицаю высодку на луну американцами если что.
Аноним 30/08/23 Срд 02:07:46 763952 400
>>763951
"Как в анекдоте" тут как раз ты, долбоёб. Сколько по-твоему длится лунный день?

>Вообще, не важно кто на какой орбите. Это слабо зависит от удаленности.
Это в первую очередь зависит от удалённости.

>У спутников Юпитера еще большие разрывы между собой по температуре
Например?
Аноним 30/08/23 Срд 04:14:27 763956 401
>>760570
Сьебал нахуй отсюда чушка в обср в загон. Иначе сейчас за политику обоссаным отсюда полетишь
Аноним 30/08/23 Срд 06:20:32 763959 402
>>763947
>температура поверхности и температура атмосферы разные вещи, дебс. американцы не босиком гуляли, а в толстенных ботинках скафандра.
И сколько ты погуляешь по 70 градусной каменной хуйне, когда у тебя ПРОБЛЕМЫ С ОХЛАЖДЕНИЕМ (это одна из основных проблем в космосе, если что), а всё, что можно сделано из металла, особенно оборудование, луноходы и спусковая капсула. Да ещё и всё обёрнуто золотой фольгой для удержания тепла - все ведь думали, что на Луне мороз и лёд в те годы
Аноним 30/08/23 Срд 11:30:02 763967 403
Почему БОГ сделал такую низкую дальность прорисовки вселенной?
Я уничтожу половину вселенной свет дойдет только через миллион лет
Аноним 30/08/23 Срд 11:32:59 763968 404
>>763959
>Да ещё и всё обёрнуто золотой фольгой для удержания тепла - все ведь думали, что на Луне мороз и лёд в те годы
Ебать ты неуч, охуеть вообще.
Аноним 30/08/23 Срд 11:33:18 763969 405
>>763967
Вычислительные мощности не бесконечные
Аноним 30/08/23 Срд 14:29:29 763977 406
>>763946
>Ну тогда получается, что на Марсе тоже высокие температуры, нет?
Там Солнце пожиже светить и нет специфического космического выветривания.
>Ну кроме полюсов. Хотя на Луне почему то температура оказалась высокая 70 градусов. Сколько же тогда в районе лунного экватора?
Не такая уж высокая. Неожиданностью было в том, что реголит на месте посадки прогневается Солнцем значительно глубже, чем в других местах. На экваторе температура реголита за 110 может заходить.
>Все равно нихуя не понимаю, американцы гуляли по Луне и не замечали какая высокая температура вокруг. И что раньше никто не додумался замерить температуру?
Реголит это очень маленькие пылинки не правильной формы, часто наэлектризованы, отчего площадь контакта с другими поверхностями очень маленькая. Поэтому они очень плохо передают тепло простой теплопередачей, в добавок у них низкая удельная теплоемкость. Даже если реголит был температурой в 300 градусов, особой разницы не было. Солнце в таких условиях сильнее нагревать будет.
>А если задумался, то какого хрена температура в 70 градусов вызывает удивление?
То что тамошний реголит сильно отличается от других реголит по оптическим свойствам и это не было видно с орбиты.
Аноним 30/08/23 Срд 14:38:56 763978 407
>>763977
Ну вот это уже более-менне нормальное объяснение. Короче, дело тупо в реголите.
Так короче, теперь на пальцах поясните, чем реголит на луне сильно отличается от грунта марса? Или, блядь, реголит на Луне, пиздос какой уникальный и больше нигде не встречается?
>даже если реголит был температурой в 300 градусов, особой разницы не было. Солнце в таких условиях сильнее нагревать будет.
И все равно, а какая же будет температура не реголита на поверхности, предметов на поверхности хотя бы там от 1,5 метра от поверхности?
Аноним 30/08/23 Срд 15:11:32 763979 408
>>763978
Реголит на Луне сильно восстановленный. Железо и титан там встречаются в низких степенях окисления, а иногда вообще в элементарном виде. Это происходит из-за постоянной бомбардировкой поверхности Луны протонами из солнечного ветра.
Включения металлических частиц железа и его сплавов может доходить больше 1% по массе. Эта включения и и делают реголит очень темным. Реголит только из пироксена и плагиоклаза светлый, отдельные его кристаллы прозрачны.
У Марса есть хоть какая-то атмосфера, которая блокирует солнечный ветер и следовательно комическое выветривание. Реголит на Марсе сильно окисленный, там даже перхлораты присутствуют. В добавок основной компонент марсианского реголита это гидратированные минералы - грубо говоря глины, точнее соленная замороженная глина.
Лунный реголит в некотором смысле уникальный, в том смысле, что это бывший кусок мантии Земли, а не просто рандомное силикатное говно, оставшиеся после формирования Солнечной системы. У астероидов состав и свойства другие.

>И все равно, а какая же будет температура не реголита на поверхности, предметов на поверхности хотя бы там от 1,5 метра от поверхности?
Сильно зависит от поверхности самих предметов, поверхности внизу, как светит солнце и есть ли у предметов другие источники тепла.
Если просто металлическая болванка, то ее температура будет около 40-50 градусов при условие, что Солнце в зените.
Аноним 30/08/23 Срд 18:25:43 763988 409
>>763979
Спасибо за развернутый ответ, вроде ты шаришь.
Аноним 31/08/23 Чтв 13:54:59 764007 410
>>763832
Постоянная Хаббла как бэ говорит о том, что Вселенная не сужается.... никогда...
Аноним 31/08/23 Чтв 13:56:44 764009 411
Правда ли что пишут что шар для боулинга имеет больше шероховатостей и перепадов высот чем планета земля? Даже учитвая всякие марианские впадины и эвересты.
Аноним 31/08/23 Чтв 13:57:16 764010 412
>>763867
Гравитационные сили больше, чем ректаливные силы излучения Хокинга
> за горизонты совместных событий?
Куда - куда, простите?
Аноним 31/08/23 Чтв 13:57:45 764011 413
>>763867
Гравитационные сили больше, чем ректаливные силы излучения Хокинга
> за горизонты совместных событий?
Куда - куда, простите?
Аноним 31/08/23 Чтв 14:00:45 764013 414
>>763868
при любых
относительно чего угодно
можешь измерять время в секундах - так проще....
Аноним 31/08/23 Чтв 14:02:22 764014 415
>>763873
Перестань мучить лягушек, долбодятел! найди себе тян.... притягательную...
Аноним 31/08/23 Чтв 14:03:22 764015 416
Аноним 31/08/23 Чтв 14:06:04 764016 417
show102.png 6Кб, 270x120
270x120
>>763887
Спасибки!
правда я все равно ни черта не понял...
А какие существуют гиперсферы в реальном мире?
Аноним 31/08/23 Чтв 14:06:38 764017 418
>>763917
или не пальцем....
Аноним 31/08/23 Чтв 14:08:27 764018 419
>>763917
А перистальтика сама работает, или тоже упихивают?
Аноним 31/08/23 Чтв 14:19:27 764020 420
>>763867
Фотоны, как носители информации, никогда не покидают окрестностей чд в пределах радиуса Шварцшильда?
Да.
Вероятно, они и из точки сингулярности не покидают. Масса фотона, однако.
Аноним 31/08/23 Чтв 14:20:41 764021 421
>>763880
>Это примудал один выживший из ума шиз
В 18 веке, и уже тогда было понятно, что это хуйня полная.
Аноним 31/08/23 Чтв 14:21:34 764022 422
>>764020
Да нет там никакой сингулярности.
Аноним 31/08/23 Чтв 16:14:15 764026 423
насколько нагреется жопа и когда она охладится если пернуть в открытом космосе?

как долго понос будет высыхать если обосраться?
Аноним 31/08/23 Чтв 16:52:52 764027 424
>>764026
Пердежь в вакууме это адиабатическое расширение газа, и твоя жопа будет охлаждаться, а не нагреваться.

Дрисня высохнет довольно быстро, за несколько минут.
Аноним 31/08/23 Чтв 17:28:12 764028 425
>>764027
ты это сам придумал, конч?
Аноним 31/08/23 Чтв 19:07:13 764031 426
image.png 324Кб, 903x960
903x960
Аноним 01/09/23 Птн 00:38:03 764034 427
как происходит стыковка члена и жопы в условиях невесомости?
Аноним 01/09/23 Птн 11:36:37 764047 428
>>764022
А что же там? Зловещий хохот Эйнштейна?
Аноним 01/09/23 Птн 11:38:07 764048 429
>>764034
Какой АйКью у спросившего? :)
Аноним 01/09/23 Птн 12:48:42 764057 430
>>764048
134. я заплатил за тест 10к, так что это достоверный результат.
Аноним 01/09/23 Птн 13:21:48 764064 431
>>764057
134 - 10к = 124?
Ты же понимаешь, что, доплатив, ты получаешь бонус к интеллекту, разве нет?
Аноним 01/09/23 Птн 13:49:20 764065 432
>>764034
Проведи испытания в условиях, приближенных к (имитирующих) невесомость.
Например, в бассейне.
(помните анекдот про двух геев в бассейне?)
Аноним 01/09/23 Птн 14:04:44 764069 433
>>764065
знаю анекдот про моисеева и пенкина в ванне, а про пидоров в басейне - нет.
Аноним 01/09/23 Птн 14:07:36 764070 434
>>764018
Сама работает - это рефлекс. А если не работает, то человека раскручивают за волосы и еда под действием центробежной силы пролазит в желудок. Сечас ты спросишь почему за волосы, а не за руки крутят?
Аноним 01/09/23 Птн 14:09:58 764071 435
>>760386 (OP)
Хочу на Луну! Как проехать "зайцем"?
Аноним 01/09/23 Птн 14:11:32 764072 436
>>764071
Такто на луну летают, а не ездят.
Аноним 01/09/23 Птн 14:14:20 764073 437
show.png 8Кб, 270x120
270x120
>>764072
Как полететь на Луну, если в кармане дюдя свистит?
совсем свистит
Аноним 01/09/23 Птн 14:15:44 764074 438
>>764072
Так проложите дорогу, в чем проблема-то? Асфальта мало, что ли?
Аноним 01/09/23 Птн 14:17:43 764075 439
>>764073
Так это в одну сторону полёт
>>764074
вращение земли и луны не синхронно, дорога оторвётся
Аноним 01/09/23 Птн 14:20:00 764076 440
>>764075
>дорога оторвётся
А вы нормально сделайте дорогу! Кчественно! Надежно! И ничего у вас не оторвется!
Аноним 01/09/23 Птн 14:23:53 764078 441
>>764076
Да хоть суперкачественно и пиздато сделать, из бескислородного асфальта покрытого золотым серебром, один хуй порвётся. Луна весит слишком много.
Аноним 01/09/23 Птн 14:27:45 764079 442
>>764078
А сделать резиновую веревочную лестницу? Растяжимую?
Аноним 01/09/23 Птн 14:35:03 764080 443
>>764079
Она не сможет бесконечно растягиватся. Да и охуеешь лезьти до луны (точнее до отметки где гравиташка заметно ослабнет), я лазил по лестнице на 120 метров так что поверь.
Аноним 01/09/23 Птн 14:37:28 764081 444
>>764080
Если со скоростью метр в секунду, то за 11 лет вполне можно долезть.
Аноним 01/09/23 Птн 14:59:59 764083 445
>>764081
А в космосах как дышать будеш? Шланг до земли растянешь? Прикинь сколько он весить будет, хотя бы 10 км шланга 16 мм, охуеешь протягивать через него воздух, хотя может капилярный эфект на первых порах сработает. Надо будет тащить насос, кабель к нему. Жрать и пить ты тоже захочешь наверно, отдыхать, спать и срать в висячем положении, ещё скафандор. Самое хуёвое - когда будешь ползти на высоте орбиты мкс, космачи с мкс тебя засмеют, назовут мудаком и будут фотать как ты срёш в скуфандр и показывать на тебя пальцем.
Аноним 01/09/23 Птн 15:09:01 764086 446
>>764080
>бесконечно растягиватся
почему бесконечно? разве там расстояние максимально не 400000 км (примерно)?
Аноним 01/09/23 Птн 15:26:19 764089 447
>>764083
Робот Фёдор долезет.
Аноним 01/09/23 Птн 15:27:37 764090 448
Аноним 01/09/23 Птн 15:41:43 764092 449
>>764086
Я ж написал - вращение не синхронизировано, луна вокруг земли почти месяц летит, а земля за 24 часа поворачивается. Смекаешь? А если делать лесницу не прикреплённую к земле - то будешь гонятся за ней по всей планете с ебовейшей скоростью. Не нашёл нигде трассу орбиты луны, к сожалению, чтоб ты увидел маршрут свисающего конца лестницы. Плюс если лесница не растягиваемая, то её длина будет как до луны в перигее, тоесь близко к земле она будет не постоянно, изза прецессии точка перигея меняется и лестница может быть над океаном, сам перигей может быть то выше то ниже.
Вобщем оставь затею с лестницей.
Аноним 01/09/23 Птн 16:09:53 764096 450
>>764092
Так вроде это рабочая идея, давно уже предложена
Аноним 01/09/23 Птн 16:13:42 764098 451
>>764096
А кто предложил, Незнайка на пару с Чебурашкой?
Аноним 01/09/23 Птн 16:19:36 764099 452
Это примерно длина дорог Великобритании, ничего фантастического. Робот Фёдор за 11 лет по всем дорогам вполне себе пройдёт. И даже скуф в скуфандре.
Аноним 01/09/23 Птн 18:58:34 764105 453
>>764016
>А какие существуют гиперсферы в реальном мире
В смысле, в нашём трёхмерном где найти четырёхмерный объект? В математике и на некоторых трёхмерных картинках и анимациях наложениями
Аноним 01/09/23 Птн 20:58:52 764106 454
>>764105
Электрон же вполне четырех мерный объект, электромагнитное поле тоже.
Аноним 01/09/23 Птн 22:48:38 764110 455
>>764064
Не, там рандом. Платишь за тест и у тебя результат рандомно либо выше, либо ниже, чем был прежде. Но это не потому что результаты подменивают, а потому что по факту оплаты ты немножко меняешься и становишься умнее или тупее.
Аноним 01/09/23 Птн 22:50:39 764111 456
>>764070
Нет, я спрошу - за какие волосы крутят?
Аноним 01/09/23 Птн 22:51:53 764112 457
>>764076
Дорогу намотает на земляшку что стукнет об нее луну.
Таким образом ты действительно создашь бесперелетный доступ к луне, браво!
Аноним 02/09/23 Суб 08:21:35 764123 458
>>764112
>Дорогу намотает на земляшку что стукнет об нее луну.
На рельсы поставьте стык дороги с земляшкой. И рельсы по экваторы проложите
Аноним 02/09/23 Суб 10:14:52 764130 459
>>764106
Хуйню не неси. Электрон - это элементарная частица. Элементарные частицы - точечные объекты, они никак "четырехмерными" быть не могут
Аноним 02/09/23 Суб 10:35:40 764131 460
>>764130
Легко могут, почему не могут-то?
Аноним 02/09/23 Суб 12:07:18 764138 461
>>764130
Электрон описывается четырех компоннетной волновой функцией.
Эти четыре числа невписываются в привычное нам пространство-время, но их вращение имеют определенную связь с вращением в пространстве-времени. Это позволяет переходить между пространством-временем и пространство электронов, но с некотороц неоднозначностью. А ещё это позволяет естественным образом определить спин и позитрон.
Аноним 02/09/23 Суб 16:16:53 764166 462
>>764131
Потому что это точечные объекты
>>764138
Это математические построения, а не реальное четырехмерное пространство. Мы живем в трехмерии
Аноним 02/09/23 Суб 18:02:01 764174 463
>>764166
>Потому что это точечные объекты
Это не объяснение, точки могут быть в сколько угодно измерениях.
Аноним 02/09/23 Суб 19:39:05 764188 464
>>764166
> точечные объекты
у точечных объектов не может быть спина
Аноним 03/09/23 Вск 09:01:39 764218 465
Вопрос скорее из области биологии, но так как навеян этим тредом то высру тут. Нужен ли живым существам, включая нас естественный радиационный фон ? Если мы создадим среду без него, то повлияет ли это на здоровье/продолжительность жизни в лучшую/худшую сторону ?
Аноним 03/09/23 Вск 10:01:02 764220 466
>>764174
Электрон существует в нашей Вселенной. Наша Вселенная трехмерна
>>764188
Так спин и не является буквальным вращением вокруг своей оси. Элементарные частицы - это не крутящиеся шарики
Хочу написать Сайфай Аноним 03/09/23 Вск 13:16:02 764238 467
Меня интересует вопрос с длиной года планеты. Полный оборот вокруг звезды она совершает за 118 земных дней. Планета должна быть обитаемой и полностью покрытой океаном. Волнует, какая звезда больше подойдёт для такой планеты? И не попадёт ли мирок в приливный захват?
Аноним 03/09/23 Вск 15:39:28 764245 468
>>764218
Гумилёв считал, что да. Радиация способствует мутациям микроорганизмов, что приводит к распространению новых штаммов, в том числе и поражающих человека. Иногда единственным симптомом является потеря страха смерти. Приспособившиеся к наличию таких больных народы вырезают мирных культурных соседей, например, Евросоюз. В этом суть истории человечества.
Аноним 03/09/23 Вск 18:57:45 764254 469
>>764238
Планета такая возможна, и этот будет пограничный случай красного и оранжевого карлика.
Землеподобные планеты рядом у таких звезд редкость. Скорее будет суперземля с первичной атмосферой. С океанам есть большая проблема, либо это жалкая лужица с ультра плотной атмосферой плотнее чем на венере, либо многокилометровой пиздец с фазовым переходов воды в лед.
Приливный захват то, не будет, но лун при такой близости к планет не будет.
Аноним 04/09/23 Пнд 06:23:15 764278 470
почему случаются вещи вроде разбитой луны 25?
Аноним 04/09/23 Пнд 07:42:24 764279 471
>>764278
В каком плане "вещи вроде"?

Почему АМС бьются? Потому что трудно делать технически сложные системы вдали от фактического места эксплуатации и без права на ошибку.

Почему АМС летают раз в несколько десятилетий и не происходит перехода неудачного опыта в удачный? Потому что Попыту весь этот космос-хуёсмос только для размахивания по телевизору нужен, и этап накопления неудачного опыта для такого не подходит — не в принципе не подходит, а на фоне достижений других стран. Проще заказывать в СМИ восторженные статьи о ядерном буксире до бесконечности.
Аноним 04/09/23 Пнд 11:58:38 764296 472
>>760822
>В таком случае давай разберем по частям идею с вертолетами.
Вертолеты должны быть черные. Иначе ни как. Думаешь инженерчики не просчитывали идеи воздушного старта?
Аноним 04/09/23 Пнд 13:10:36 764306 473
>>760969
У тебя просто отца нормального ни когда не было который бы тебе вырезал из дерева игрушку в виде птички.
Аноним 04/09/23 Пнд 13:53:31 764309 474
>>761619
Потрясён людской беспечностью?
Лизни прохладный клитор бесконечности
По-злому клинит на ерунде?
Дай за щеку чёрной дыре
Аноним 04/09/23 Пнд 14:25:17 764310 475
Анон, а какие вообще обоснования теории большого разрыва? Разве ослабление гравитации со временем когда-то наблюдалось? И Галактики же не меняют размер в расширяющейся вселенной, с чего вообще взяли что все разорвет в будущем?
Аноним 04/09/23 Пнд 16:25:14 764321 476
>>764310
>Анон, а какие вообще обоснования теории большого разрыва?
Куча их, за полным список иди в википедию.
Главное это наблюдательные данные. Нас окружает громадное количество галактики, которые от нас разбегаются. Если интерполировать все в зад, то они в измеримом прошлом должны были быть прямо рядом тут.
>Разве ослабление гравитации со временем когда-то наблюдалось?
Гравитация никогда не ослабевала, менялся только масштабный фактор.
>И Галактики же не меняют размер в расширяющейся вселенной
Тащемта очень даже меняют. Часто очень далекие галактики по угловым размерам почти такие или больше галактик, которые находятся посередине между первыми и нами.
>с чего вообще взяли что все разорвет в будущем?
Это зависит от уравнения состояния вселенной, точнее от баланса гравирующей материи и темной энергии, а так же ее природы. Большой разрыв это крайний сценарий, где темная энергия бескомпромиссно доминирует на материей
Аноним 04/09/23 Пнд 18:27:34 764328 477
>>764310
> обоснования
челик который эту хуйню форсил просто засунул в уравнение состояния красивую циферку типа ~15 лярд лет назад бахнуло, го подставим такое чтоб через соизмеримое случился конец

чистейшей воды маргинальщина, и при нём оно было сильно дохуя меньше, да и тренд уточнений идёт на уменьшение и примерно аппроксимируется к некоторому константному расширению
типа максимальный разлёт галактик, но никакого разрыва молекул/атомов

и даже если не константа, тепловая смерть случится на много порядков раньше
Аноним 04/09/23 Пнд 21:09:08 764338 478
>>764321
>Куча их, за полным список иди в википедию.
Википедию пишут пидарасы. Я и сам частенько её правлю, так что я им не доверяю. Ты лучше своими словами распиши, чему считаешь ТББ верной? Анон всё же ближе петушиной вики.

>Гравитация никогда не ослабевала
А гравитация вообще равномерна? Свойства пространства равномерны?

>Часто очень далекие галактики по угловым размерам почти такие или больше галактик, которые находятся посередине между первыми и нами.
Значит далекие галактики не такие уж и далекие

>темная энергия бескомпромиссно доминирует на материей
Как энергия может существовать самостоятельно?
Аноним 04/09/23 Пнд 23:21:58 764343 479
>>764279
> Почему АМС бьются? Потому что трудно делать технически сложные системы вдали от фактического места эксплуатации и без права на ошибку.
двадцать лет было, ленивые каксакиры из поккокмока могли все просчитать и даже больше

> Почему АМС летают раз в несколько десятилетий и не происходит перехода неудачного опыта в удачный? Потому что Попыту весь этот космос-хуёсмос только для размахивания по телевизору нужен, и этап накопления неудачного опыта для такого не подходит — не в принципе не подходит, а на фоне достижений других стран. Проще заказывать в СМИ восторженные статьи о ядерном буксире до бесконечности.
храйкни
Аноним 05/09/23 Втр 11:11:55 764348 480
>>764310
>какие вообще обоснования теории большого разрыва?
Не существует такой теории
>Разве ослабление гравитации со временем когда-то наблюдалось?
Это что за хуйня такая - "ослабление гравитации"? Никакого "ослабления гравитации" нет и о нем никто в науке не говорит. Ты сам придумал эту несуразную поебень или вычитал где?
>И Галактики же не меняют размер в расширяющейся вселенной
Они и не должны, потому что галактики - гравитационно связанные системы
>с чего вообще взяли что все разорвет в будущем?
Ни с чего. Так называемый большой разрыв - довольно маргинальный сценарий будущего Вселенной. Всерьез его из специалистов по теме мало кто рассматривает. Так что им можно полностью пренебречь. Вселенную ждет не разрыв очка, а вечное ускоренное расширение и тепловая смерть
Аноним 05/09/23 Втр 12:10:22 764350 481
>>764254
А если планета где-то в полтора раза больше Марса?
Аноним 05/09/23 Втр 15:34:44 764363 482
>>764348
Тепловая смерть означает, что где-то в крайне отдалённом будущем в результате флуктуации опять случится большой взрыв? Или большой разрыв этому помешает?
Аноним 05/09/23 Втр 20:01:37 764371 483
>>764350
Атмосферу рядом с звездой не удержит, и никаких тебе океанов.
Аноним 05/09/23 Втр 21:49:39 764379 484
>>764350
Больше по объёму, массе, диаметру? Разные же вещи, ёпта.
Аноним 05/09/23 Втр 22:26:46 764380 485
1518613248327.jpg 870Кб, 2560x1600
2560x1600
Тёмной энергии не существует. Расширение вселенной происходит за счёт массы материи, которая растягивает пространство, т.е. вселенная не расширяется, а растягивается пространство. Время напрямую связанно с пространством, потому, как является тем самым процессом натяжения пространства.
Аноним 06/09/23 Срд 02:52:19 764390 486
>>764380
>вселенная не расширяется, а растягивается пространство.
блять это и называют расширением вселенной
Аноним 06/09/23 Срд 02:56:14 764391 487
>>764310
ебать бред сумасшедшего
Аноним 06/09/23 Срд 04:17:10 764392 488
Посоны, что там недавно об Юпитер ёбнулось ? Лень гуглить.
Аноним 06/09/23 Срд 06:40:36 764396 489
Аноним 06/09/23 Срд 07:18:21 764399 490
А какие обоснования у тепловой смерти вселенной? По первому закону термодинамики энергия же не исчезает в никуда. При большом взрыве вещество должно было иметь максимальную энтропию и гравитация успешно снизила её. И раз уж водород существует, то как-то и где-то он образовывается. Если бы все вещество вселенной образовалось исключительно при большом взрыве мы бы имели одну черную дыру вместо вселенной, это же очевидно.
Аноним 06/09/23 Срд 08:23:12 764402 491
image.png 101Кб, 400x320
400x320
>>764399
Если вселенная из нихуя появилась, то почему бы нахуй и без следа не исчезнуть?
Аноним 06/09/23 Срд 08:52:44 764404 492
>>764392
ОБ юпитер нельзя ёбнутся, можно В юпитер ёбнутся.
Аноним 06/09/23 Срд 08:54:30 764405 493
>>764402
Ну так а обоснования какие? Возможно ли уничтожить вещество или энергию чтобы от неё ничего не осталось?
Аноним 06/09/23 Срд 09:36:18 764407 494
>>764405
Из чего вселешка появилась? Из нихуя. Значит это её фундаментальное свойство. Что из нихуя появилось - может и исчезнуть в нихуя
Аноним 06/09/23 Срд 09:53:34 764408 495
913eea5783c8a4e[...].jpg 27Кб, 404x404
404x404
>>764407
Ну то есть теоретических обоснований у тепловой смерти ровно столько же сколько у большого разрыва. Литералли НИХУЯ, но при этом большой разрыв это выдумки идиотов, а тепловая смерть 100% предсказание.
Аноним 06/09/23 Срд 09:58:44 764409 496
>>764363
>означает, что где-то в крайне отдалённом будущем в результате флуктуации опять случится большой взрыв?
Нет. Это означает, что в отдаленном будущем погаснут и умрут все звезды, остынут все нейтронные звезды и белые карлики (они станут черными), испарятся все черные дыры и во Вселенной не останется нихуя, кроме болтающихся в пустоте на космологических расстояниях друг от друга потухших нейтронных звезд и черных карликов. Ну и еще какие-нибудь отдельные электроны и нейтрино будут летать
>>764380
Это тред тупых вопросов, а не тупых утверждений. На хуй отсюда
>>764399
>Если бы все вещество вселенной образовалось исключительно при большом взрыве мы бы имели одну черную дыру вместо вселенной, это же очевидно
Очевидно лишь, что ты нихуя не понимаешь в теме, на которую пытаешься здесь пиздеть.
Что ты вообще подразумеваешь под Большим взрывом?
Аноним 06/09/23 Срд 10:36:18 764410 497
Как и за счёт чего мы можем положительно утверждать, что перед нами та или иная планета - хтноческая, а не изначально мелкая каменная?
Аноним 06/09/23 Срд 10:43:23 764411 498
>>764409
>Очевидно лишь, что ты нихуя не понимаешь в теме, на которую пытаешься здесь пиздеть.
>Что ты вообще подразумеваешь под Большим взрывом?
Большой взрыв это расширение пространства.
Сам представь все вещество вселенной, то которое сейчас в ней имеется, в ограниченном пространстве на начальных этапах существования вселенной. Там притяжение между веществом было бы как в черной дыре, оно просто не смогло бы расширятся вслед за пространством. Следовательно вещество возникло чуть позже расширения пространства, сразу за ним, а значит неизвестный пока процесс создания вещества все таки присутствует в нашей вселенной.
То есть водород вполне себе генерируется неким процессом, который так же генерировал его в начале существования вселенной.
Аноним 06/09/23 Срд 14:35:05 764433 499
>>764379
Плотность планеты, как у Земли.
Радиус - 4212 км.
Масса - 0,268 земной
Аноним 06/09/23 Срд 14:36:49 764434 500
Как бы скорее всего выглядел нейтроний, если бы его удалось извлечь и посмотреть на него (разумеется до того, как его распидорасит от декомпрессии)
Аноним 06/09/23 Срд 15:11:35 764438 501
>>764411
>вещество возникло чуть позже расширения пространства
Что значит "чуть позже расширения пространства"? Пространство всегда расширялось и продолжает расширяться.
Вещество во Вселенной появилось в конце инфляционной стадии, в результате вакуумных флуктуаций. А не схлопнулось в черную дыру, потому что темпы расширения были гигантскими. Впрочем, локальные флуктуации плотности вполне могли приводить к образованию ЧД - их называют первичными
Аноним 06/09/23 Срд 15:56:49 764442 502
>>764438
А что находится там, где нет пространства? Как это вообще "нет пространства"?
Аноним 06/09/23 Срд 16:53:23 764452 503
>>764442
Да есть оно везде. Разница только в количестве материи на квадратный метр. Но нашим мозгам такое понимание пока не доступно. Как это так епть пустота и все... и бесконечность...
Аноним 06/09/23 Срд 17:08:36 764453 504
>>764452
Но долбаки утверждают что пространства до большого взрыва не было. А, блядь, объяснить, что было вместо пространства не могут, но пространства не было. Ёбаный их рот.
Аноним 06/09/23 Срд 20:26:05 764461 505
>>764453
Играл в Портал? Представь что ты меж двух бесконечно огромных порталов, которые могут свободно двигаться, и порталы начали движение друг к другу. Когда между порталами останутся сантиметры ты раздавишь сам себя. А вот вселенная это пространство между этими порталами. Порталы раздвигаются-вселенная расширяется. И для наблюдателя родившегося внутри межпортального пространства будет все равно что там за порталами, потому что он никогда не сможет взаимодействовать с тем миром. Может что-то есть за пределами вселенной, может что-то было до вселенной, этого никогда не узнать, поэтому лучше забить.
Аноним 06/09/23 Срд 20:40:37 764462 506
>>764461
пространство было до вселешки, оно пустое просто было. И время было, просто ничего не происходило.
Аноним 06/09/23 Срд 20:53:34 764464 507
>>764462
Это тебе твоим мясным мозгом так кажется, ты проецируешь видимое на реальное, а реальности похую на то как мясные мозги ее воспринимают.
Аноним 06/09/23 Срд 21:08:13 764466 508
>>764442
>А что находится там, где нет пространства?
Абсурдный вопрос. Из разряда "как быть там, где быть невозможно"
>Как это вообще "нет пространства"?
Никак
Аноним 06/09/23 Срд 21:19:05 764467 509
>>764442
>А что находится там, где нет пространства?
А что находится по адресу пятидесятого гигабайта на восьмигиговой плашке памяти?
Аноним 06/09/23 Срд 23:00:24 764473 510
>>764462
Чел, это та область в которой можно сколько угодно гадать, научными теориями эти гадания все равно никогда не станут. Зачем разводить тут завадомо антинаучную хуйню, для этого есть другие разделы.
Аноним 07/09/23 Чтв 02:11:03 764491 511
1359325862119.png 238Кб, 700x531
700x531
>>764396
Блядь, это твоя мамаша, а блять, это выражение такое, когда кринж от сверх тупорылости ебанатов.
Аноним 07/09/23 Чтв 05:50:51 764501 512
>>764491
Выражение тоже пишется через "д". Через "т" пишут неграмотные уёбки.
Аноним 07/09/23 Чтв 09:39:58 764505 513
>>764491
>а блять, это выражение такое, когда кринж от сверх тупорылости ебанатов
Это ты сам придумал, чурка пиздоглазая? Напустили сюда всяких чучмеков без знания русского языка, пиздец
Аноним 11/09/23 Пнд 06:54:58 764725 514
Почему на Российском сегменте МКС не пьют воду из мочи
Аноним 11/09/23 Пнд 12:36:04 764741 515
>>764725
Это типо зашкварно. Хотя каждая молекула воды на Земле уже не раз побывала и в чьем-то пинусе и в чьем-то анусе.
Аноним 11/09/23 Пнд 13:15:08 764743 516
>>764725
Придерживаются понятий. Это наши традиционные ценности.
Аноним 11/09/23 Пнд 22:09:06 764754 517
image.png 920Кб, 2350x641
2350x641
image.png 949Кб, 1871x647
1871x647
image.png 757Кб, 2262x649
2262x649
image.png 845Кб, 2222x660
2222x660
Kurzgezagt выпустил видео про ядерную войну. Ну ладно, у них вроде подобное направление было.
Atomic Frontier выпустил видео про ядерную войну. Вопреки названию, это не в его стиле.
Veritasium выпустил видео про ядерную войну.
С чего это три несвязанных попсойсай канала вдруг выпускают видео про ядерную войну? Блядь, ОНИ ЗНАЮТ что ли, и это зог сигнализирует умным людям? Но я не знаю что делать!
инб4 ты не умный, челадой маловек не для вас написано
инб5 иди в зогач - не пойду, там как раз наддуют паранои и убедят что на самом деле надо к ядерному холокосту готовиться.
Аноним 12/09/23 Втр 05:07:59 764761 518
>>764725
Из мочи они варят самогон, пьют сами и продают пендосам
Титановолхв, взываю к тебе Аноним 12/09/23 Втр 12:29:34 764774 519
a19c9a09d8dcc54[...].jpg 43Кб, 719x996
719x996
Если на Титане слабая гравитация и атмосфера плотнее и толще земной, значит ли это, что Титан получится покинуть на винтовом самолёте?
Аноним 12/09/23 Втр 13:18:51 764775 520
>>764754
А может быть, это всё каналы-миллионники, которые следят за трендами и пытаются вскочить на хайптрейн. Может такое быть? У них у всех и по киберпанку одновременно вышли.
Аноним 12/09/23 Втр 17:43:16 764785 521
>>764774
Нет, на винтовом самолет нет.
Из-за низкой температуры в атмосфере очень низкая скорость звука, а винтом выше скорости звука не полетаешь.
Впрочем с прямоточным реактивном двигатели вполне даже можно до второй космической разогнаться.
Аноним 12/09/23 Втр 19:19:45 764788 522
>>764775
>следят за трендами
Так это и хотел узнать, что случилось что вдруг тренд на ЯО пошел? Кто-то новую бонбу изобрел что ли?
>У них у всех и по киберпанку одновременно вышли.
По-моему, ни у одного из упомянутых каналов не было ничего по киберпанку.
Аноним 12/09/23 Втр 19:43:49 764789 523
Аноним 12/09/23 Втр 19:47:37 764790 524
>>764788
У кургезаца про оцифровку сознания - это коллаб с поляками, а у веритасмума по-моему тоже что-то было. Могу ошибаться. Я это вообще к другому.
Просто тема всплыла. В мире вон что происходит. Плохие новости скучные, а жуткие темы смотрят (помнишь ту серию из Смешариков, "Последняя радуга"?).
А верить, что это проделки зога - прямой путь в дурку.
Аноним 12/09/23 Втр 20:19:51 764791 525
>>764790
Ладно, спасибо за ответ.
>А верить, что это проделки зога - прямой путь в дурку.
Я когда попытался туда попасть, они когда узнали что я двачер, сказали идти на хуй, они будут заниматься теми кому хотя бы можно помочь.
Я знаю что не зог, просто не понял с чего это вдруг зачесались на эту тему. Если еще и PBS SpaceTime про это видео снимет, я охуею.
Аноним 13/09/23 Срд 10:50:44 764803 526
>>764774
Нет.
Первая космическая скорость (v1)1,867 км/с
Вторая космическая скорость (v2) 2,639 км/с
Винтовые так быстро не летают.
Аноним 13/09/23 Срд 12:04:14 764808 527
image.png 116Кб, 1375x567
1375x567
Аноним 14/09/23 Чтв 13:27:07 764836 528
>>764123
Вау, а это идея! осталось только найти инвестора
Аноним 14/09/23 Чтв 13:28:33 764838 529
>>764433
Ни хуя она тебе не удержит, братан
Аноним 14/09/23 Чтв 13:35:42 764839 530
>>764410
Какой млкий хтонический грмматика, каменно - плнетарная просто
Аноним 14/09/23 Чтв 13:46:13 764841 531
>>764434
Никак. Буквально. Как газ. Ну посмотри на гелий в воздушном шарике.
Кстати, если тебе и удасться его извлечь (наверное, срезав его с нейтронной звезды, - например, струной Орфея или волосом с хвоста единорога), - он просто провалится сквозь твою ладонь, сквозь пол, прямо к центру плоской Земли... (тяжесть, однако)
Реквестирую испрвления и уточнения, потому что вопрос - то интересный....
Аноним 14/09/23 Чтв 13:49:27 764842 532
>>764442
> А что находится там, где нет пространства? Как это вообще "нет пространства"?
Пространство - это не что- то "само по себе существующее", вроде речки для рыбок без рыбок.
Это, во- первых, пространство-время, четырехмерное.
Ну что может произойти за 0.000000000000001 секунд? А за 0?
Аноним 14/09/23 Чтв 13:51:15 764843 533
>>764467
Если нет кода ошибки, прочитаешь содержимое буфера чтения.
Наверное.
Аноним 14/09/23 Чтв 13:51:48 764844 534
>>764467

Если нет кода ошибки, прочитаешь содержимое буфера чтения.
Наверное.
Аноним 14/09/23 Чтв 13:53:17 764845 535
>>764462
Не было.
Пространство и время, блять, относительно.
Если нету двух точек, то и пространства (между ними) нету.
Аноним 14/09/23 Чтв 14:09:53 764846 536
Аноним 14/09/23 Чтв 14:17:22 764847 537
show.png 6Кб, 270x120
270x120
>>764453
Твоя ошибка в том, что ты предполагаешь, будто пространство когда - либо БЫЛО.
А его, блять, не было. И нет. И не будет. Иллюзия это. Эйнштейн доказал.
>>764442
>Как это так епть пустота и все... и бесконечность...
бесконечность, между чем и чем, мать твою?
Если нет крайних точек, то что?
пространство == расстояние, болван. Нет "между" == нет пространства
Смирительную рубашку этим двачерам!
ну и капча
Аноним 14/09/23 Чтв 20:22:01 764871 538
>>764844
>>764843
Ну вот и в телескоп ты может что-то "там" увидишь, да.
Может вообще там как китайская терабайтная флешка будет.
Аноним 14/09/23 Чтв 20:22:57 764872 539
>>764841
> он просто провалится сквозь твою ладонь, сквозь пол, прямо к центру плоской Земли... (тяжесть, однако)
>Реквестирую испрвления и уточнения
Уточняю, никуда он не провалится, т.к. не успеет по простой причине его моментального распидорашивания.
Аноним 14/09/23 Чтв 21:12:24 764873 540
>>764872
А как с вариантом кварковой материи? Во многих моделях большие куски должны быть стабильные, а иногда даже энергетически предпочтительнее обычной материи.
Что же будет если возьмем страпельку в руки?
Аноним 14/09/23 Чтв 21:26:51 764875 541
image.png 216Кб, 533x300
533x300
Аноним 15/09/23 Птн 06:11:56 764885 542
>>764873
Дедпула в фильме отстрапелили, например.
Аноним 15/09/23 Птн 06:17:27 764886 543
>>764842
Причём тут вообще время? В войдах милионы лет нихуя не происходит, но это не значит что для них время не идёт.
Аноним 15/09/23 Птн 07:20:00 764889 544
>>764885
Интересный факт: на самом деле Раян Рейнолдс хотел чтобы Морена Бакарин его пежнула, и даже записал это на видево, но режиссер нашел это видео и решил включить в фильм. А Раян и не против.

>>764886
>В войдах милионы лет нихуя не происходит
Происходит же. Там все равно частиц навалом.
Аноним 15/09/23 Птн 12:33:51 764894 545
Аноним 15/09/23 Птн 23:42:00 764918 546
image.png 236Кб, 600x418
600x418
Войд Волопаса: что скрывает самое страшное место Вселенной?
Аноним 16/09/23 Суб 00:16:28 764922 547
Аноним 16/09/23 Суб 00:34:07 764926 548
>>764922
А как из ничего появилось НИЧЕГО?
Аноним 16/09/23 Суб 00:54:14 764930 549
>>764926
Непонятно как тебе ответить.
И что ты ожидаешь примерно услышать-то?
Что-то вроде "квантовые флюктуации после бв случайно говна недоложили в этот регион"?
Если ты что-то из области кефира ожидаешь, то ты не по адресу.
Говори сразу, если ты из этих, чтобы разговор не продолжать попусту.
Общение с такими просветленными никогда не задается.
Аноним 16/09/23 Суб 10:35:58 764952 550
>>764918
Да хули там скрывать-то? Там просто меньше вещества, чем в среднем по Вселенной. Вот тебе и весь "войд". Ничего сверхъестественного. Ктулху там не живет
Аноним 17/09/23 Вск 17:27:23 765025 551
image.png 275Кб, 1006x515
1006x515
Аноним 18/09/23 Пнд 15:47:02 765125 552
>>764918
А как мы можем быть уверенны, что там, собственно, войд, а не, например, облако межгалактического газа звездочки нам закрывает, абидно?
Ответить в тред Ответить в тред

Check this out!

Настройки X
Ответить в тред X
15000
Добавить файл/ctrl-v
Стикеры X
Избранное / Топ тредов